Success in Scie P2 G.C.E (2015 - 2023)

You might also like

Download as pdf or txt
Download as pdf or txt
You are on page 1of 103

SUCCESS IN SCIENCE

SUCCESS IN SCIENCE P2, 2015 – 2023 G.C.E DON’T POST / SHARE IT ON SOCIAL MEDIA

PAPER 2 (G.C.E) (CHEMISTRY) VOLUME TWO

2ND EDITION – OCTOBER 2023


Instant Revision from E.C.Z Question Papers
GRADE 10 – 12 2015 – 2023

THIS SCIENCE P2 (CHEMISTRY) BOOKLET WILL HELP YOU TO:


 Revise E.C.Z Past Papers with answers instantly
 Self-Check Examination Questions
 Know the Top Examiner’s Mind
 Pass exam easily
POSTING / SHARING THIS BOOKLET ON SOCIAL MEDIA IS ILLIGAL
THEREFORE, GET IT FROM A RIGHTFUL OWNER MUSONDA LAURENT
AT K20 IN SOFTCOPY OR K80 IN HARDCOPY

CALL /TEXT MUSONDA LAURENT 0954482350


WhatsApp: 0954482350 / 0965038377 / 0974794056 OR
EMAIL: laurentmsnd@gmail.com
SUCCESS ONLY THE BEST IS GOOD ENOUGH
Page 1 of 103 WhatsApp: 0954482350 / 0965038377 / 0974794056 OR EMAIL: laurentmsnd@gmail.com
LAURENT MUSONDA Call / Text: 0954482350
SUCCESS IN SCIENCE P2, 2015 – 2023 G.C.E DON’T POST / SHARE IT ON SOCIAL MEDIA

Selling any success booklet in softcopy by anybody is illegal. Therefore, get it from the rightful
owner Musonda Laurent in softcopy at k20.
CALL / TEXT: 0954482350
WhatsApp: 0954482350 / 0965038377 / 0974794056
Email: laurentmsnd@gmail.com
Terms and Conditions
Any person having any success booklet have agreed that:
1 He / She will NOT be Posting, sharing, transmitting or circulating any success booklet on
social media.
2 He / She will NOT be Selling any success booklet in softcopy.
NOTE: Selling in hardcopy is allowed, hence, people are allowed to print and photocopy
any success booklet without changing / erasing any part of ITS writing.
Anybody who will be found wanting in any Terms and Conditions above will take responsibility
of the consequences.
GET ANY SUCCESS BOOKLET IN SOFTCOPY AT K20 FROM THE RIGHTFUL OWNER MUSONDA LAURENT

PRIMARY: GRADE 5 – GRADE 7 JUNIOR SECONDARY: GRADE 8 – GRADE 9


1. ENGLISH (2015 – 2022) 1. ENGLISH P2 (2015 – 2022)
2. MATHS (2015 – 2022) 2. MATHS P1 (2015 – 2022)
3. INTEGRATED SCIENCE (2015 – 2022) 3. MATHS P2 (2015 – 2022)
4. SOCIAL STUDIES (2015 – 2022) 4. INTEGRATED SCIENCE (2015 – 2022)
5. C.T.S (2015 – 2022) 5. SOCIAL STUDIES (2015 – 2022)
6. SPECIAL PAPER 1 (2015 – 2022) 6. BUSINESS STUDIES (2015 – 2022)
7. SPECIAL PAPER 2 (2015 – 2022) 7. COMPUTER STUDIES (2015 – 2022)
8. RELIGIOUS EDUCATION (2015 – 2022 EXT)
SENIOR SECONDARY: GRADE 10 – GRADE 12
1. BIOLOGY P1 (2015 – 2022) G.C.E 18. PRINCIPLES OF ACCOUNT P1 (2015 – 2021)
2. BIOLOGY P1 (2015 – 2022) INTERNAL 19. PRINCIPLES OF ACCONT P2 (2015 – 2022)
3. BIOLOGY P2 (2016 – 2022) G.C.E 20. PURE CHEMISTRY P1 (2015 – 2021)
4. BIOLOGY P2 (2015 – 2022) INTERNAL 21. PURE CHEMISTRY P2 (2015 – 2021)
5. COMMERCE (2020 – 2022) 22. PURE PHYSICS P1 (2015 – 2021)
6. COMPUTER P1 G12 (2017 – 2022) 23. PURE PHYSICS P2 (2015 – 2021)
7. CIVIC EDUCATION (2017 – 2022) 24. RELIGIOUS EDUCATION 2046 (2016 – 2022)
8. ENGLISH P2 (2015 – 2022) G.C.E 25. RELIGIOUS EDUCATION 2044 (2020 – 2022)
9. ENGLISH P2 (2015 – 2022) INTERNAL 26. SCIENCE P1 (PHYSICS) (2016 – 2022) G.C.E
10. GEOGRAPHY P1 (2015 – 2021) 27. SCIENCE P1 (PHYSICS) (2015 – 2022) INTERNAL
11. GEOGRAPHY P2 (2015 – 2022) 28. SCIENCE P2 (CHEMISTRY) (2015 – 2022) G.C.E
12. HISTORY P1 (2019 – 2022) 29. SCIENCE P2 (CHEMISTRY) (2016 – 2022) INTERNAL
13. HISTORY P2 (2019 – 2022)
14. MATHS P1 (2017 – 2022) G.C.E
15. MATHS P1 (2015 – 2022) INTERNAL
16. MATHS P2 (2017 – 2022) G.C.E
17. MATHS P2 (2016 – 2022) INTERNAL

Page 2 of 103 WhatsApp: 0954482350 / 0965038377 / 0974794056 OR EMAIL: laurentmsnd@gmail.com


LAURENT MUSONDA Call / Text: 0954482350
SUCCESS IN SCIENCE P2, 2015 – 2023 G.C.E DON’T POST / SHARE IT ON SOCIAL MEDIA
Centre Candidate
Number Number
Candidate name: ____________________________

EXAMINATIONS COUNCIL OF ZAMBIA


Examination for General Certificate of Education Ordinary Level
Science 5124/3
(Chemistry)
PAPER 3

Thursday 20 AUGUST 2015

SECTION A [45 MARKS]


Answer all the questions.
Write your answers in the spaces provided on a question paper.
1. The diagram below shows a method by which a mixture of sodium chloride and iodine
crystals can be separated.

(a) Give the name of the separation technique shown in the the diagram above.
…………………………………………………………………………………………………………………………….[1]
(b) Name apparatus A.
…………………………………………………………………………………………………………………………[1]
(c) Identify solid B
…………………………………………………………………………………………………………………………[1]
(d) Explain why sodium chlrode solution is regaeded as a mixture.
…………………………………………………………………………………………………………………………[1]
2. Study the nuclide of a sodium isotope below and answer the questions that follow; 24
11Na.

Page 3 of 103 WhatsApp: 0954482350 / 0965038377 / 0974794056 OR EMAIL: laurentmsnd@gmail.com


LAURENT MUSONDA Call / Text: 0954482350
SUCCESS IN SCIENCE P2, 2015 – 2023 G.C.E DON’T POST / SHARE IT ON SOCIAL MEDIA

(a) What is meant by the term nucleon number?


………………………………………………………………………………………………………………………………
[1]
(b) The table below describes the composition of the nucleus of sodium. Complete the
table by writing the total number of each particle in one atom of the element.

Name of particle Total number in one atom


Neutron
Protons
Electron

(c) Write the formula of sodium ion.


……………………………………………………………………………………………………………………….. [1]
(d) Write the electronic configuration of the sodium ion.
………………………………………………………………………………………………………………………… [1]

3. Study the flow diagram below carefully.


Process A Substance B Process C Ethanol
Heavy oil

(a) Name
(i) Process A …………………………………………………………………………………………
(ii) Process C ………………………………………………………………………………………….. [2]

(b) Name Substance B.

……………………………………………………………………………………………………………………….. [1]
(c) Give one significance of Process A.

…………………………………………………………………………………………………………………………. [1]
4. Lead oxides are environmental pollutants.
(a) Name the source of these pollutants.
…………………………………………………………………………………………………………………………
(b) What is their major adverse effect on humans?
…………………………………………………………………………………………………………………………
(c) State the measures that have been put in place in order to avoid this kind of
pollution
…………………………………………………………………………………………………………………………
(d) (i) Explain the meaning of the term global warming.
……………………………………………………………………………………………………………..
(ii) Name a pollutant which causes global warming.
……………………………………………………………………………………………………………..

Page 4 of 103 WhatsApp: 0954482350 / 0965038377 / 0974794056 OR EMAIL: laurentmsnd@gmail.com


LAURENT MUSONDA Call / Text: 0954482350
SUCCESS IN SCIENCE P2, 2015 – 2023 G.C.E DON’T POST / SHARE IT ON SOCIAL MEDIA

5. The diagram below illustrates the reaction that occurs when a small piece of
potassium metal is dropped into cold water mixed with a little red litmus solution.

(a) The potassium metal reacts explosively with cold water'. Predict the
expected colour change of the litmus solution.
………………………………………………………………………………………………………….
(b) Give a reason for the colour change observed in (a) above.
………………………………………………………………………………………………… [1]
(c) Bubbles of a colourless gas are produced during this reaction.
(i) Name the gas ………………………………………………………………………. [1]
(ii) Describe the identity test of this gas
……………………………………………………………………………………………….. [1]
(d) Construct a balanced chemical equation for the reaction between cold
water and potassium metal.
………………………………………………………………………………………………………..

6. Pure dry crystals of magnesium sulphate can be made by reacting excess magnesium
powder with dilute sulphuric acid.
(a) (i) Explain why excess magnesium powder is used.
……………………………………………………………………………………………………….
(ii) How is excess magnesium powder removed from the reaction mixture?
……………………………………………………………………………………………………………….
(b) The chemical equation for the reaction between magnesium and sulphuric
acid is given below.
Mg(s) + H2SO4 (aq) MgSO4 (aq) + H2 (g)
Calculate the mass of hydrogen gas produced when 96g of magnesium reacts
with dilute sulphuric acid.

7. Bromine is in Group VII of the Periodic Table. It reacts with aqueous


potassium iodide to form potassium bromide and iodine.
(a) Describe what you would see when bromine gas is bubbled through
aqueous potassium iodide.
………………………………………………………………………………………………………………
(b) Construct a balanced chemical equation for the reaction between bromine
and aqueous potassium iodide.
……………………………………………………………………………………………………………….
(c) Name the type of reaction in (b) above:
…………………………………………………………………………………………………………..
(d) Explain why iodine does not react with a solution of potassium bromide.

Page 5 of 103 WhatsApp: 0954482350 / 0965038377 / 0974794056 OR EMAIL: laurentmsnd@gmail.com


LAURENT MUSONDA Call / Text: 0954482350
SUCCESS IN SCIENCE P2, 2015 – 2023 G.C.E DON’T POST / SHARE IT ON SOCIAL MEDIA

…………………………………………………………………………………………………………….
(e) State the similarity in the electron arrangement of halogens.
…………………………………………………………………………………………………………….

8. Aluminium, copper, sodium and zinc are a few of the common metals.
Use only these metals to answer the questions below.
(a) Arrange the metals above in order of decreasing reactivity.

…………………………………………………………………………………………………………………………. [1]
(b) (i) Select the metal which is obtained from its ore by reduction with carbon.
…………………………………………………………………………………………………………… [1]
(ii) Name the ore from which the metal in b (i) is obtained.

……………………………………………………………………………………………………………….[1]
(c) Explain why aluminium is unreactive even though it is high in the reactivity series.

…………………………………………………………………………………………………………………………. [2]

9. A student carried out an experiment in which two gas jars were set up as shown.

(a) Describe what was observed when the glass disc was removed.
…………………………………………………………………………………………………………………………………

(b) Explain why this change occurred.


………………………………………………………………………………………………………………………………
………………………………………………………………………………………………………………………………

Page 6 of 103 WhatsApp: 0954482350 / 0965038377 / 0974794056 OR EMAIL: laurentmsnd@gmail.com


LAURENT MUSONDA Call / Text: 0954482350
SUCCESS IN SCIENCE P2, 2015 – 2023 G.C.E DON’T POST / SHARE IT ON SOCIAL MEDIA

SECTION C [20 Marks]


Answer any two questions.
10. (a) Name the black solid reagent that can be reacted with dilute sulphuric
add to obtain Copper (II) sulphate solution.
(b) Describe how you can prepare Copper (II) sulphate crystals.
[4]
(c) Construct a balanced chemical equation with state symbols for the reaction
in (a) above.
[2]
(d) In another reaction Zinc oxide was reacted with dilute sulphuric acid
according to the equation below.
ZnO (s) + H2SO4 (aq) + 6H2O (l) ZnSO4.7H2O
Calculate the mass of zinc sulphate heptahydrate, ZnSO 4.7H2O, crystals that
can be formed from 19.6g of sulphuric acid.
(a) Define water of Crystallisation.

11. Explain the meaning of


(a) (i) a filtrate
(ii) a residue [2]
(b) Describe two ways of separating a solid from a liquid in a suspension. [4]
(c) (i) State one difference between simple distillation and fractional distillation.
[2]
(ii) Name two industries in Zambia where fractional distillation is used. [2]

12. Propanoic acid, CH3CH2COOH, is a weak acid.


(a) Explain what is meant by the term weak acid.
(b) Propanoic acid reacts with potassium carbonate. Write a balanced chemical
equation for this reaction.
(c) Calcium reacts with propanoic acid to form calcium propanoate and hydrogen gas
as shown below.
Ca (s) + 2CHCH2CO2H (aq) ----------> (CH3CH2CO2)2Ca (aq) + H2 (g)
Calculate the mass of hydrogen produced in the reaction shown above when
calcium is added to excess propanoic acid.
(d) Terylene has the simplified structure shown below.

(i) State the functional groups on the monomers used to make Terylene.
(ii) State the type of polymerisation that occurs when Terylene is made.
(e) Many problems are caused by the disposal of plastics. Describe one method of
disposal of plastic and a problem caused by this method.

Page 7 of 103 WhatsApp: 0954482350 / 0965038377 / 0974794056 OR EMAIL: laurentmsnd@gmail.com


LAURENT MUSONDA Call / Text: 0954482350
SUCCESS IN SCIENCE P2, 2015 – 2023 G.C.E DON’T POST / SHARE IT ON SOCIAL MEDIA

EXAMINATIONS COUNCIL OF ZAMBIA


Examination for General Certificate of Education Ordinary Level
SCIENCE 5124/1
(CHEMISTRY PART)
PAPER 1 Multiple Choice

Monday 11 JULY 2016 1HOUR

1 The proof that matter is made up of tiny particles which cannot be seen is studied in
A Wave motion B Brownian motion. C Random motion. D Particle motion.

2 The diagram below shows an arrangement of apparatus used to separate a mixture of


methanol boiling point 15OC and water boiling point 1000C.

Which part of the apparatus is wrongly arranged and what is likely to be liquid J when
corrected?
Correction Liquid J
A Heat the condenser Water
B Cold water should enter from bottom and leave through the top. Methanol
C No heating of the mixture is required Methanol
D Cold water should enter from the bottom and leave through the Water
top.

3 The element with a symbol X has an atomic number 12 and another element Y has atomic
number 7. What is the formula of the compound formed between elements X and Y?
A XY3 B X2Y3 C X3Y2 D X3Y

Page 8 of 103 WhatsApp: 0954482350 / 0965038377 / 0974794056 OR EMAIL: laurentmsnd@gmail.com


LAURENT MUSONDA Call / Text: 0954482350
SUCCESS IN SCIENCE P2, 2015 – 2023 G.C.E DON’T POST / SHARE IT ON SOCIAL MEDIA

4 What is the mass of water produced when 58g of butane (C4H10) burns completely in air
according to the equation below?
2C4H10 + 13O2 -------------> 8CO2 + 10H2O
A 90g B 80g
C 68g D 60g

5 Potassium hydroxide reacts with phosphoric acid as follows:


3KOH + H3PO4 -----------> _________ + 3H2O
A K2PO4 B K3PO4
C K4PO4 D K4PO4

6 The chemical equation below illustrates the complete combustion of ethanol in air.
W C2H5 OH + X O2 -------------> Y CO2 + Z H2O
Find the values of W, X, Y and Z in the above chemical equation.
W X Y Z
A 2 6 4 6
B 1 3 2 3
C 2 4 4 6
D 1 1 2 3

7 A few drops of universal indicator were added to a soil sample solution. The indicator
turned yellow. What is the nature of the soil sample?
A Acidic B Basic
C Neutral D Salty

8 How does the atomic structure of an element relate to the Periodic Table?
A valency shell determine the group for the element.
B The valency electrons determine the period for the element.
C Number of electron shells determines the group for the element.
D Number of electron shells determines the period for the element.

9 The preparation of lead (II) sulphate salt is best done by using which chemical equation?
A Pb(S) + H2SO4 (aq) -------> PbSO4(s) + H2(g)
B PbO(S) + H2SO4 (aq) -------> PbSO4(s) + H2O(g)
C Pb(NO3)2(aq) + H2SO4 (aq) -------> PbSO4(s) + 2HNO3(aq)
D PbCl2(S) + Na2SO4 (aq) -------> PbSO4(s) + 2NaCl(aq)

10 The substance which dissolves in water to form a solution that changes universal indicator
to blue is called...
A an acid anhydride. B a basic salt.
C an amphoteric oxide. D an alkali.

Page 9 of 103 WhatsApp: 0954482350 / 0965038377 / 0974794056 OR EMAIL: laurentmsnd@gmail.com


LAURENT MUSONDA Call / Text: 0954482350
SUCCESS IN SCIENCE P2, 2015 – 2023 G.C.E DON’T POST / SHARE IT ON SOCIAL MEDIA

11 Which of the following statements is the correct characteristic of the elements found
between group II and group III of the Periodic Table?
A Their oxides dissolve in water leaving an alkaline solution.
B Their compounds usually form coloured solutions.
C They all displace hydrogen from dilute sulphuric acid.
D Their oxides are thermally stable.

12 Element T floats on water and reacts violently with cold water. Identify element T.
A Aluminum
B Barium
C Carbon
D Potassium

13 Dilute hydrochloric acid solution was added to substance X and effervescence was observed.
The gas was bubbled through solution Y and the solution formed a white suspension. What
is the identity of substance X and solution Y?
Substance X Solution Y
A Carbonate Calcium hydroxide
B Metal Universal indicator
c Carbonate Calcium carbonate
D Nitrate Litmus solution

14 Choose a set which corresponds to a metal and the main ore from which it is obtained.
Metal Mineral ore
A Zinc Bauxite
B Copper Bauxite
C Iron Magnetite
D Aluminium Magnetite

15 The correct description of non-metals is that they...


A react with acids to form acid salts.
B oxidize by loss of electrons.
C form basic oxides when they react with oxygen.
D form acidic oxides when they react with oxygen.

16 The correct description for the distinguishing test between alkane and alkenes is that ...
A alkenes quickly decolourise aqueous bromine, while the alkane do not.
B alkanes quickly decolourise aqueous bromine, while the alkenes do not.
C alkanes react with steam while alkenes do not.
D alkanes undergo polymerisation, while alkenes do not.

Page 10 of 103 WhatsApp: 0954482350 / 0965038377 / 0974794056 OR EMAIL: laurentmsnd@gmail.com


LAURENT MUSONDA Call / Text: 0954482350
SUCCESS IN SCIENCE P2, 2015 – 2023 G.C.E DON’T POST / SHARE IT ON SOCIAL MEDIA

17 Halogens belong to Group VII of the Periodic Table, and have a valency of one because they ...
A have seven electrons each.
B ionise by gain of one electron.
C ionise by loss of one electron.
D all exist as diatomic molecules in nature.

18 Polymers are very cheap plastics and are widely used by mankind. What is the best way of
disposing them off?
A Burning them completely in air B Creating dumping sites for them
C Leaving them to rot on land D Bury them

19 The major pollutant on the Copperbelt comes from the copper smelters.
Which of the following information gives the correct identity of the pollutant and its adverse
effect of the environment?
Identity Adverse effect
A Sulphur Forms acid rain
B Sulphur dioxide Causes bronchitis
c Nitrogen dioxide Causes bronchitis
D Carbon dioxide depletes the ozone layer

20 The table below gives information on the suitable monomer, polymer formed and its use.

Which set of information above is correct?

Page 11 of 103 WhatsApp: 0954482350 / 0965038377 / 0974794056 OR EMAIL: laurentmsnd@gmail.com


LAURENT MUSONDA Call / Text: 0954482350
SUCCESS IN SCIENCE P2, 2015 – 2023 G.C.E DON’T POST / SHARE IT ON SOCIAL MEDIA

Centre Candidate
Number Number
Candidate Name_________________________

EXAMINATION COUNCIL OF ZAMBIA


Examination for General Certificate of Education Ordinary Level

Science 5124/3
Paper 3 (chemistry)

Tuesday 12 JULY 2016

Time: 1 hour 15 minutes


SECTION A [45 Marks]
Answer all questions in this section.

1. All matter is made up of small particles which are referred to as the basic units of matter.
The small particles of matter are always in random motion.
(a) State any two basic units of matter.
………………………………………………………………………………………………………………………… [2]
(b) Suggest two pieces of evidence which show that matter is made up of tiny, randomly
moving particles.
…………………………………………………………………………………………………………………………. [2]
(c) The diagram below represents one of the physical states of matter.

Name this physical state of matter and explain why it has a fixed volume.
Name: ………………………………………………………………………………………………………………
Explanation: …………………………………………………………………………………………………….. [2]

2. Each of the following statements is incorrect. But one scientific term is replaced by a correct
one, the statement then becomes true. Identify the wrong word (term), underline it and in
the space provided write the correct word.
(a) Condensation is the change of state of matter from solid to liquid.
……………………………………………………………………………………………………………………… [1]
(b) The basic units of matter that exist in sodium chloride are molecules.
………………………………………………………………………………………………………………………… [1]

Page 12 of 103 WhatsApp: 0954482350 / 0965038377 / 0974794056 OR EMAIL: laurentmsnd@gmail.com


LAURENT MUSONDA Call / Text: 0954482350
SUCCESS IN SCIENCE P2, 2015 – 2023 G.C.E DON’T POST / SHARE IT ON SOCIAL MEDIA

(c) Isotopes are compounds with the same molecular formula but different structural
formulae. ………………………………………………………………………………………………………… [1]
(d) Calcium hydroxide is classified as an amphoteric substance because it dissolves in
sodium hydroxide solution.
…………………………………………………………………………………………………………………………. [1]
(e) Zinc and chlorine bond covalently.
…………………………………………………………………………………………………………………………. [1]

3. Two elements A and D are represented by the notations shown below.

(a) Draw electron-shell diagrams to show the arrangement of electrons in the atoms of
A and D [2]
Atom of element A Atom of element D

(b) (i) When atoms of element A and atoms of element D react together, what
type of bonding occurs?
Type of bonding: ………………………………………………………………………………… [1]
(ii) Using the dot and cross diagram, show the structure of the compound
formed between A and D showing all the electron shells. [2]

(c) State any two physical properties of the compound formed between A and D.
(i) …………………………………………………………………………………………………………………….
(ii) …………………………………………………………………………………………………………………… [2]

4. When carbon dioxide gas is bubbled through aqueous calcium hydroxide (lime
water), the following reaction occurs:
Ca(OH)2 (aq) + CO2 (g) CaCO3 (s) + H2O (l)
(a) Write a word equation for this reaction.
………………………………………………………………………………………………………………
(b) If 1.2dm3 of carbon dioxide gas measured at r.t.p was bubbled through
excess calcium hydroxide solution. Calculate the:
(i) Mass of calcium hydroxide solution which reacted. [2]

(ii) Mass of calcium carbonate formed. [2]

(iii) Describe what would be seen as the carbon dioxide gas is bubbled
through aqueous calcium hydroxide.
…………………………………………………………………………………………………… [2]

Page 13 of 103 WhatsApp: 0954482350 / 0965038377 / 0974794056 OR EMAIL: laurentmsnd@gmail.com


LAURENT MUSONDA Call / Text: 0954482350
SUCCESS IN SCIENCE P2, 2015 – 2023 G.C.E DON’T POST / SHARE IT ON SOCIAL MEDIA

5. The following table shows some information about the second member in the respective
homologous series. Complete the table by filling in the correct information.
homologous series formula Name of the member Displayed full structural formula
Alkane
Ethanol

6. The diagram below shows part of the Periodic Table.

State the name of an element from this part of the Periodic Table which
(a) Forms univalent cations.
……………………………………………………………………………………………………………. [1]
(b) Reacts most vigorously with cold water.
……………………………………………………………………………………………………………… [1]
(c) Forms oxides of formulae XO and X 2O, where X represents the chemical
symbol of the element.
……………………………………………………………………………………………………………
(d) Does not react with oxygen.
…………………………………………………………………………………………………………. [1]
7. Solutions of hydrochloric acid and sodium hydroxide are mixed together and they
react as shown below.

(a) Write a balanced chemical equation for this reaction.


………………………………………………………………………………………………………… [1]
(b) State the name for this type of reaction and write an ionic equation for it.
Type of reaction ………………………………………………………………………….. [2]
Ionic equation ………………………………………………………………………………
(c) Describe how crystals of sodium chloride can be obtained from the solution
formed during the reaction. [2]

Page 14 of 103 WhatsApp: 0954482350 / 0965038377 / 0974794056 OR EMAIL: laurentmsnd@gmail.com


LAURENT MUSONDA Call / Text: 0954482350
SUCCESS IN SCIENCE P2, 2015 – 2023 G.C.E DON’T POST / SHARE IT ON SOCIAL MEDIA

8. When hot magnesium metal is introduced in gas jar containing carbon dioxide gas, the carbon
dioxide is decomposed and the following reaction occurs.
2Mg (s) + CO2 (s) MgO(s) + C (s)
In this reaction, both oxidation and reduction occur.
(a) What name is used to describe reaction in which both reduction and oxidation occur?
…………………………………………………………………………………………………………………
(b) State, with reason, which substance is oxidised and which substance is reduced.
(c) (i) Substance oxidised: …………………………………………………………………………………
Reason: …………………………………………………………………………………………………
(ii) Substance reduced: ……………………………………………………………………………………
Reason: ………………………………………………………………………………………………………
(d) Which substance is reducing agent?
…………………………………………………………………….……………………………………………………
(e) Classify the two types of oxides from the equation above.
(i) …………………………………………………………..………………………………………………
(ii) ………………………………………………………………………………………………………

Section B
[20 MARKS]
Answer any two questions
9. Copper is extracted from one of the ores, copper pyrites, CuFeS2. The copper pyrites is first
converted to copper (I) sulphide before copper metal is produced.

(a) Write down the equations leading to the formation of copper metal from copper
pyrites.

(b) Explain why this extraction leads to air pollution. [1]

(c) State the process used to purify copper. [1]

(d) State any two large scale uses of copper and give reasons why copper is used in that
way.

(e) (i) Copper occurs 'native' in some countries such as Zambia.


Explain what is meant by the word 'native'. [1]

(ii) Name one alloy made from copper metal. [1]

Page 15 of 103 WhatsApp: 0954482350 / 0965038377 / 0974794056 OR EMAIL: laurentmsnd@gmail.com


LAURENT MUSONDA Call / Text: 0954482350
SUCCESS IN SCIENCE P2, 2015 – 2023 G.C.E DON’T POST / SHARE IT ON SOCIAL MEDIA

10. When a mixture of ammonium sulphate and sodium hydroxide is heated, the reaction
represented by the word equation below occurs.
Ammonium sulphate + sodium hydroxide  sodium sulphate + water + ammonia.

(a) Write a balanced chemical equation for the above reaction. [2]

(b) Describe a chemical test you can carry out to show that ammonia is formed. [2]

(c) On a large scale, ammonia is produced by the Haber process:


(i) What are the reactants used in the Haber process? [2]
(ii) Write a balanced chemical equation for the Haber process. [2]
(iii) State the essential conditions used in order to obtain an economical yield of
the ammonia gas. [2]

11. Propene (CH3 - CH = CH2) reacts with steam to give a major product propan -2 -ol.

(a) State the essential conditions in this reaction. [2]

(b) State the kind of reaction that occurs between propene and steam. [1]

(c) However, there is also a minor by-product formed in such a reaction. [1]

(i) Give the full (displayed) structural formula and the systematic name of this
by-products. [2]

(ii) How is the major product separated from the minor product? [1]

(iii) If 2.1kg of propene is fully reacted with steam. Calculate the mass of
propan -2-ol that would be formed

(d) State the general formula for alkenes. [1]

Page 16 of 103 WhatsApp: 0954482350 / 0965038377 / 0974794056 OR EMAIL: laurentmsnd@gmail.com


LAURENT MUSONDA Call / Text: 0954482350
SUCCESS IN SCIENCE P2, 2015 – 2023 G.C.E DON’T POST / SHARE IT ON SOCIAL MEDIA

Centre Candidate
Number Number

Candidate Name_________________________

EXAMINATION COUNCIL OF ZAMBIA


Examination for General Certificate of Education Ordinary Level

Science 5124/2
Paper 2

Tuesday 1 AUGUST 2017

Time: 2 hours Marks: 85

Section A [20 marks]


Answer all questions on the answer grid provided.

A1. Which of the following is not true about evaporation?


A. It involves a physical change of state C. The particles gain kinetic energy
B. It is non-reversible change D. It weakens the intermolecular forces of
attraction

A2. The nuclide of aluminium ion is written as 27 3+


13Al , state the numbers of neutrons and electrons
in the nuclide of the ion.
Neutrons Electrons
A. 27 13
B. 14 13
C. 14 10
D. 27 10.

A3. Some cold water is poured into a conical flask and a bung inserted. The diagram shows the flask
after being left in open air for some time.

Page 17 of 103 WhatsApp: 0954482350 / 0965038377 / 0974794056 OR EMAIL: laurentmsnd@gmail.com


LAURENT MUSONDA Call / Text: 0954482350
SUCCESS IN SCIENCE P2, 2015 – 2023 G.C.E DON’T POST / SHARE IT ON SOCIAL MEDIA

What is occuring in the flask?


A. Boiling and condensation C. Evaporation and condensation
B. Evaporation and freezing D Freezing and melting

A4. Identify a mixture of substance that can be separated using the apparatus below.

A mixture of …
A. Paraffin and water C. common salt and iodine solution
B. Sugar solution and alcohol D. alcohol and water

A5. Ethane has the structure shown below.

How many of the electrons in a molecule of ethane are not involved in bonding?
A. 4 B. 3 C. 2 D. 0

A6. Which of the following is an exothermic reaction?


A. The reaction between hydrogen and iodine C. photosynthesis
B. Development of photographs D. Rusting

A7. The diagram shows an apparatus used to measure the percentage composition of
gases in the atmosphere. Phosphorous reacts with oxygen in the air to form
phosphorous (V) oxide which dissolves in water. The initial volume of gas in the tube is
80 cm3.

What volume of gas remained after all the phosphorous had burned?
A.16 cm3 B. 40 cm3 C. 60 cm3 D.64 cm3

Page 18 of 103 WhatsApp: 0954482350 / 0965038377 / 0974794056 OR EMAIL: laurentmsnd@gmail.com


LAURENT MUSONDA Call / Text: 0954482350
SUCCESS IN SCIENCE P2, 2015 – 2023 G.C.E DON’T POST / SHARE IT ON SOCIAL MEDIA

A8. Determine the relative molecular mass of lead (IV) chloride, PbCI 4.
A. 249 B. 278 C. 349 D. 378

A9. Which change will not increase the rate of a chemical reaction? An increase in ,..
A. concentration- of aqueous reactants.
B. pressure of gaseous reactants.
C. temperature of a reaction system.
D. the particle size of solid reactants.

A10. Choose a substance which when added in excess to acidic soil will increase its pH
without making it alkaline.
A. CaCl2 B. CaCO3 C. CaO D. Ca(OH)2

A11. An acid differs from a base in that an acid ...


A. Turns a red litmus paper blue. B has a pH value above 7.
C. has a sour taste. D. turns a blue litmus paper red
.
A12. Which set of elements exist as diatomic molecules at room temperature?
A. Hydrogen, oxygen, helium. B. Nitrogen, chlorine, neon.
C. Nitrogen, oxygen, fluorine. D. Oxygen, chlorine, helium.

A13. Two elements are in the same group of the periodic table.--Which property will be the
same for both elements?
A. Their boiling points B. The number of shells
C. Their electronic structure D. The charge on their ions

A14. When the temperature of a chemical reaction is increased, the kinetic energy of
particles increases and the….
A. number of effective collisions increases.
B. number of effective collisions decreases.
C. particles become far apart from each other.
D. particles become closer to each other.

A15. Which metal is extracted from its ore by reduction of its oxide by carbon?
A. Aluminum B. Copper C. Sodium D. Zinc

A16. Identify the substance which undergoes decomposition because of high temperature in the
blast furnace?
A. Calcium silicate B. Calcium carbonate C. Coke D. Slag

Page 19 of 103 WhatsApp: 0954482350 / 0965038377 / 0974794056 OR EMAIL: laurentmsnd@gmail.com


LAURENT MUSONDA Call / Text: 0954482350
SUCCESS IN SCIENCE P2, 2015 – 2023 G.C.E DON’T POST / SHARE IT ON SOCIAL MEDIA

A17. A colourless. gas can only be collected using the method shown below

What does this tell you about the gas? It is...


A. denser than air and insoluble in water. B. denser than air and soluble in water.
C. less dense than air and insoluble in water. D. less dense than air and soluble in
water.

A18. When ethene is bubbled through aqueous bromine, the solution turns
A. brown. B. colourless. C. purple. D. red.

A19. Methane is a greenhouse gas, which process releases methane into the air?
A. Combustion of petrol B. Decay of vegetable matter
C. Volcanic activity D. Photosynthesis

A20. When the temperature of a chemical reaction is increased, the kinetic energy of particles
increases and the …
A. number of effective collisions increases.
B. number of effective collisions decreases.
C. particles become far apart from each other.
D. particles become closer to each other.

Section B [45 marks]


Answer all questions in this section.
Write your answers in the spaces provided on the question paper.
B1 A spillage of 15.5 tonnes of sulphuric add results from an accident of a road tanker.
Slaked lime is used to neutralise the acid according to the equation below:
H2SO4 (aq) + Ca(OH)2 (s) CaSO4(s) + H2O (l)
(a) Balance the equation above.
………………………………………………………………………………………..[2]
(b) Determine the relative formula mass of Ca(OH) 2.
……………………………………………………………………………………………..[1]
(c) Use the balanced equation to determine the mass of calcium sulphate formed
during the neutralization of the spilt acid.
……………………………………………………………………………………………[1]
(d) Calcium hydroxide is a base, which ion present in the compound is responsible for its
basic properties?
…………………………………………………………………………………………… [1]

Page 20 of 103 WhatsApp: 0954482350 / 0965038377 / 0974794056 OR EMAIL: laurentmsnd@gmail.com


LAURENT MUSONDA Call / Text: 0954482350
SUCCESS IN SCIENCE P2, 2015 – 2023 G.C.E DON’T POST / SHARE IT ON SOCIAL MEDIA

B2 Most metals are not found as pure elements in the earth's crust, and iron is one such metal. Iron is
extracted from its ore in a blast furnace.
(a) Name two other raw materials added to the blast furnace other than haematite.
…………………………………………………………………………………………………..
……………………………………………………………………………………………..[2]
(b) Write a balanced chemical equation for the reduction of the iron ore to the metal.
……………………………………………………………………………………………………………………………………………….
.[2]
(c) State two conditions necessary for rusting to occur.
…………………………………………...........................................................................................................
[2]

B3 Use the following list of elements to answer the questions below. Iron, Lithium, Mercury, Oxygen,
Potassium, Sulphur. Each element can be used once, more than once or not at all. Which element
(a) Is used as a catalyst in the manufacture of ammonia in the Haber process?
……………………………………………………………………………………………. [1]
(b) Is lower than sodium in the reactivity series?
…………………………………………………………………………………………….. [1]
(c) Is a non-metallic solid, whose atoms contain only six valency electrons?
……………………………………………………………………………………………… [1]
(d) Is in Period 6 of the Periodic Table?
……………………………………………………………………………………………. [1]
(e) Forms an oxide which is amphoteric?
…………………………………………………………………………………………….. [1]

B4 The complete combustion of petrol produces carbon dioxide, water vapour and sulphur dioxide.
The exhaust gases from cars contain oxides of nitrogen.
(a) State the source of these oxides.
……………………………………………………………………………………………………. [1]
(b) The sulphur dioxide and oxides of nitrogen from cars cause an environmental problem.
(i) State what this problem is.
…………………………………………………………………………………………… [1]
(ii) What is the effect of this problem on buildings painted with lime?
……………………………………………………………………………………………. [1]
(c) Carbon monoxide is produced when there is incomplete combustion of carbon containing fuels
like petrol. Name one solid product of incomplete combustion of petrol in car engines.
…………………………………………………………………………………………………… [1]

B5 Choose a word from the box to match the chemical reactions expressed below.
Combustion Synthesis
Decomposition Displacement
Neutralisation Precipitation

Page 21 of 103 WhatsApp: 0954482350 / 0965038377 / 0974794056 OR EMAIL: laurentmsnd@gmail.com


LAURENT MUSONDA Call / Text: 0954482350
SUCCESS IN SCIENCE P2, 2015 – 2023 G.C.E DON’T POST / SHARE IT ON SOCIAL MEDIA

(a) AgNO3(aq) + HCl (aq) AgCl (s) + HNO3(aq)


…………………………………………………………………………………………………….. [1]
(b) Zn(s) + CuSO4 (aq) ZnSO4 (aq) + Cu (s)
…………………………………………………………………………………………………….. [1]
(c) H+ (aq) + OH- (aq) H2O (i)
…………………………………………………………………………………………………… [1]
(d) C (s) + O2(g) 3CO2 (g)
……………………………………………………………………………………………………. [1]
(e) NH3 (g) + HCl (aq) NH4Cl(S)
…………………………………………………………………………………………………….. [1]

B6 Study the following list of processes: melting, chemical change, sublimation, condensation,
evaporation, dissolving. Which of the processes listed above best describe what is taking place in
each of the following?
(a) The formation of water droplets on the window pane on a cold day.
…………………………………………………………………………………………………… [1]
(b) The formation of liquid sodium chloride from solid sodium chloride due to strong heating
……………………………………………………………………………………………………… [1]
(c) The formations of iodine vapour from solid iodine on heating.
……………………………………………………………………………………………………….. [1]
(d) Adding sugar to hot tea and stirring it.
…………………………………………………………………………………………………………. [1]
(e) The formation of calcium oxide when calcium is heated in the air.
…………………………………………………………………………………………………………. [1]

B7 Define the following terms:


(a) (i) Endothermic reaction
………………………………………………………………………………………… [1]
(ii) Exothermic reaction
………………………………………………………………………………………….. [1]
(b) Give an example of each type of reaction in (a) in nature.
Endothermic reaction………………………………………………………… [1]
Exothermic reaction………………………………………………………….. [1]
(c) Describe the effect of exothermic reactions in industries on the environment.
………………………………………………………………………………………………………….. [1]

B8 (a) Give two reasons why chemistry is important in industry.


……………………………………………………………………………………………………………
………………………………………………………………………………………………………[2]
(b) State any two laboratory safety rules.
……………………………………………………………………………………………………………
………………………………………………………………………………………………………[2]

Page 22 of 103 WhatsApp: 0954482350 / 0965038377 / 0974794056 OR EMAIL: laurentmsnd@gmail.com


LAURENT MUSONDA Call / Text: 0954482350
SUCCESS IN SCIENCE P2, 2015 – 2023 G.C.E DON’T POST / SHARE IT ON SOCIAL MEDIA

B9 The table below shows the properties of elements W, X, Y and Z.


Reaction with Effect of
Elements Cold Dilute heat on
water Oxygen hydrochloric their
acid carbonates
W X X X √
X X √ √ √
Y √ √ √ X
Z √ √ √ √

Key:
(√) Chemical change occurs (X) NO Chemical change occurs
Use the letters to answer the following questions.
(a) Arrange the elements in the increasing order of their reactivity.
…………………………………………………………………………………………………………
…………………………………………………………………………………………………….[1]
(b) Which element is used in the making of car bodies?
……………………………………………………………………………………………………. [1]

(c) Suggest a method by which element W can be extracted from its ore.
…………………………………………………………………………………………………… [1]
(d) Using the letter Z, construct an equation to illustrate the effect of heat on its carbonate.
……………………………………………………………………………………………………[1]
(e) State the element which is suitable for making ornaments. Explain your answer.
……………………………………………………………………………………………………[2]

Section C [20 marks]


Answer any two (2) questions from this section. Write your answers in the separate answer
booklet provided.
C1 The structure below is for a polymer.

(a) (i) Name the polymer.


(ii) Name and draw the structural formula of the monomer for the polymer. [3]
(b) (i) State the type of polymer that is shown above? [1]
(ii) Give one use of the polymer. [2]
(c) Ethanol is an alcoholic beverage which can be brewed from cassava. Outline the process by
which ethanol can be prepared. [3]
(d) Ethanol is used as a fuel. Construct a balanced chemical equation for its complete
combustion.

Page 23 of 103 WhatsApp: 0954482350 / 0965038377 / 0974794056 OR EMAIL: laurentmsnd@gmail.com


LAURENT MUSONDA Call / Text: 0954482350
SUCCESS IN SCIENCE P2, 2015 – 2023 G.C.E DON’T POST / SHARE IT ON SOCIAL MEDIA

C2 Calcium chloride, CaCl2 is a soluble salt that can be prepared in the laboratory.
(a) Suggest suitable reactants for its preparation in the laboratory.
[2]
(b) Describe how you would prepare a pure dry sample of calcium chloride in the
laboratory.
(c) Lead (II) iodide is an insoluble salt.
(i) What method can be used to prepare it?
[1]
(ii) Write an ionic equation for the reaction used in the preparation of the salt,
include state symbols.
[3]

C3 The exhaust fumes from an internal combustion engine contain the pollutant gases carbon
monoxide and nitrogen dioxide.
(a) Many vehicles have a catalytic converter fitted on their exhaust systems.
(i) Describe the chemical reactions which occur in the catalytic converter to reduce the
emissions of carbon monoxide and nitrogen dioxide. [3]
(ii) Write a balanced chemical equation for one of the reactions that occurs in the catalytic
converter.
(b) Briefly explain the effect of carbon monoxide on human beings. [3]
(c) Suggest two other ways of minimizing pollution of the air by these two gases other than using a
catalytic converter. [2]

Page 24 of 103 WhatsApp: 0954482350 / 0965038377 / 0974794056 OR EMAIL: laurentmsnd@gmail.com


LAURENT MUSONDA Call / Text: 0954482350
SUCCESS IN SCIENCE P2, 2015 – 2023 G.C.E DON’T POST / SHARE IT ON SOCIAL MEDIA

Centre Candidate
Number Number

Candidate Name_________________________

EXAMINATION COUNCIL OF ZAMBIA


Examination for General Certificate of Education Ordinary Level

Science 5124/2
Paper 2

Thursday 1 August 2019

Time: 2 hours Marks: 85

SECTION A [20 MARKS]


Answer all the questions on the answer grid provided.

A1. 50 cm3 sample of alcohol is mixed with 50 cm3 of water.

The volume of the mixture is found to be 97 cm3. Which of the following is the best
explanation for this observation?
A. Some alcohol molecules evaporated.
B. The alcohol molecules fit into the gaps between water molecules
C. Water and alcohol react to form a gas which escapes
D. Water and alcohol react to produce a salt which then dissolves

Page 25 of 103 WhatsApp: 0954482350 / 0965038377 / 0974794056 OR EMAIL: laurentmsnd@gmail.com


LAURENT MUSONDA Call / Text: 0954482350
SUCCESS IN SCIENCE P2, 2015 – 2023 G.C.E DON’T POST / SHARE IT ON SOCIAL MEDIA

A2. A mixture Q contains three compounds T, U and V whose solubility in ethanol and water are
as shown in the table below.
Solubility in
Compounds Ethanol Water
T Very soluble Sparingly soluble
U Insoluble Very soluble
V Insoluble Insoluble

How would you separate the mixture Q to obtain pure crystals of compound U?

A. Add ethanol filter dry the residue


B. Add water filter crystallize
C. Add ethanol filter add water filter crystallize
D. Add water filter add ethanol filter crystallize

A3. The chemical formula of ammonium sulphate is …


A. NH4SO4
B. (NH4)2SO4
C. (NH4)2SO3
D. NH4SO3

A4. The atoms 31 32


15P and 16S have the same …
A. Nucleon number. B. number of electrons.
C. Number of neutrons D. Number of protons

A5. Which pair of elements can combine chemically to form single covalent bonds?
A. Hydrogen and nitrogen C. Oxygen and carbon
B Potassium and nitrogen D. Sodium and chlorine

A6. Which one of the following is not an example of mineral acid?


A. Sulphuric acid B. Nitric acid
C. Hydrochloric acid D. Citric acid

A7. Lead (II) iodide is yellow insoluble salt. Which of the following should be reacted with lead
(II) nitrate solution to produce lead (II) iodide?
A. Iodine crystals B. Iodine solution
C. Copper (II) chloride solution D. Potassium iodide solution

A8. Zinc hydroxide can react with sodium hydroxide because it is ...
A. acidic. B. amphoteric.
C. basic. D. neutral.

Page 26 of 103 WhatsApp: 0954482350 / 0965038377 / 0974794056 OR EMAIL: laurentmsnd@gmail.com


LAURENT MUSONDA Call / Text: 0954482350
SUCCESS IN SCIENCE P2, 2015 – 2023 G.C.E DON’T POST / SHARE IT ON SOCIAL MEDIA

A9. Calcium burns in oxygen according to the following equation;


2Ca (s) + O2 (g) 2CaO (s)
If 5.2g of calcium burns completely, what will be the mass of calcium oxide
produced?
A . 6.8g B . 7.3g C . 7.8g D . 8.0g

A10. Barium sulphate is best prepared by …..


A . Crystallisation. B . Neutralisation.
C . Oxidation. D . Precipitation

A11. The order of the reactivity of the elements below starting from the least
reactive is .

A. X, Y, Z B. Z, X, Y
C Y, Z, X D. Y, X, Z

A12. Which of the following explains the importance of catalysts in chemical reactions?
They…
A. Enable the activation energy to be lowered.
B. Enable reactants to be consumed completely.
C. Help to conserve reactants and products.
D. offer more energy for the reaction to take place

A13. An element was described as shown in the table below. Which


description is correct about the element at r.t.p?

Number of shells valency period Group State


A 3 4 5 3 Solid
B 3 3 3 6 Gas
C 3 6 6 3 Liquid
D 3 5 3 5 solid

A14. Which of the following observations most strongly suggests that a solid element X is
a non-metal?
A. X reacts vigorously with chlorine
B. X is a conductor of electricity
C. X forms an acidic oxide.
D. X has more than one valency.

Page 27 of 103 WhatsApp: 0954482350 / 0965038377 / 0974794056 OR EMAIL: laurentmsnd@gmail.com


LAURENT MUSONDA Call / Text: 0954482350
SUCCESS IN SCIENCE P2, 2015 – 2023 G.C.E DON’T POST / SHARE IT ON SOCIAL MEDIA

A15. Which of the following sets corresponds to a metal and the main ore from which it is
extracted?
Metal Ore
A Zinc Calamine
B Iron Bronze
C Copper Magnetite
D Aluminium Haematite

A16. Nitrogen is used to produce ammonia as shown in the diagram below.

What is X?
A. Air
B. Hydrogen
C. Oxygen
D. Water

A17. Ethene, C2H4 burns completely in air to form balanced equation for the reaction?
A. C2H4 + O —> CO2 + H2O
B. C2H4 + O2 ---> CO + H2O
C. C2H4 + 2O2 ---> 2CO2 + 2H2O
D. C2H4 + 3O2 —> 2CO2 + 2H2O

A18. The organic product of the reaction between CH3OH and HCOOH is …
A. CH3OCH3
B. HCOOCH3
C. CHOCH3
D. CH3OOH.

A19. Choose the correct monomers for terylene.


A Alcohol and carboxylic acid.
B Amine and carboxylic acid.
C Ethene and diol.
D Ethene and carboxylic acid.

Page 28 of 103 WhatsApp: 0954482350 / 0965038377 / 0974794056 OR EMAIL: laurentmsnd@gmail.com


LAURENT MUSONDA Call / Text: 0954482350
SUCCESS IN SCIENCE P2, 2015 – 2023 G.C.E DON’T POST / SHARE IT ON SOCIAL MEDIA

A20. The displayed structural formulae below are of different compounds. Which ones are
isomers?

A X and Y
B V and X
C Y and W
D V and Y

Section B [45 marks]

Answer all questions in this section.


Write your answers in the spaces provided on the question paper.

B1 The table below shows some common gases and their particulars.
Gas Relative molecular mass Vapour density
Ammonia 17 8.5
Carbon dioxide 44 22
Carbon monoxide 28 14
Helium 4 2
Nitrogen 28 14

Choose from the list given, the gas (es) that best suits the following descriptions. A gas may
be chosen once, more than once or not at all.
(a) (i) The gas that diffuses fastest. [1]

(ii) The names of the gases that diffuse at the same rate. [1]

Page 29 of 103 WhatsApp: 0954482350 / 0965038377 / 0974794056 OR EMAIL: laurentmsnd@gmail.com


LAURENT MUSONDA Call / Text: 0954482350
SUCCESS IN SCIENCE P2, 2015 – 2023 G.C.E DON’T POST / SHARE IT ON SOCIAL MEDIA

(b) State the name of the gas that would diffuse faster than any other gas shown in the
table.
(c) What is the relationship between relative molecular mass of a gas and its vapour
density?
(d) Under what condition would carbon dioxide diffuse faster than ammonia both of
equal volume?

B2 The diagram below shows the preparation of liquor in a village.

(a) Name the process demonstrated above. [1]


(b) (i) Name apparatus B. [1]
(ii) If this experiment was done in the school laboratory, what name would be
apparatus A and B? [2]
(c) What is the scientific term for the liquor/liquid in the experiment? [1]

B3 Aluminium metal was reacted with aqueous copper (ii) sulphate.


(a) Construct a balanced chemical equation for the above reaction. Include state
symbols.
(b) Deduce an ionic equation from (a). [2]

B4 Below is a list of substances which are either deliquescent or hygroscopic and


some which are not.
Aluminium hydroxide, calcium oxide, sodium hydroxide, sulphuric acid, anhydrous
calcium chloride, anhydrous copper (II) sulphate, iron (III) oxide and sodium
carbonate.
(a) From the list given above, choose one substance which is
(i) Deliquescent.

(ii) Hygroscopic.

(iii) Neither deliquescent nor hygroscopic.

Page 30 of 103 WhatsApp: 0954482350 / 0965038377 / 0974794056 OR EMAIL: laurentmsnd@gmail.com


LAURENT MUSONDA Call / Text: 0954482350
SUCCESS IN SCIENCE P2, 2015 – 2023 G.C.E DON’T POST / SHARE IT ON SOCIAL MEDIA

(b) What is the difference between a deliquescent and hygroscopic substance?

(c) Some substances are said to be efflorescent. What is an efflorescent


substance?

B5 A learner reacted a certain mass of magnesium with 150cm 3 of dilute hydrochloric


acid. 240cm3 of hydrogen gas evolved at room temperature and pressure.
(a) Construct a balanced chemical equation for the reaction .

(b) Calculate
(i) The number of moles of hydrogen in 240cm 3 of the gas.

(ii) The mass of magnesium used in the experiment to produce 240cm3 of


hydrogen at room temperature and pressure.

B6 Study the two diagrams below showing different sizes of sodium metals.

(a) (i) Which diagram would the sodium react faster if put in a
beaker containing water?
(ii) Give a reason for your answer in (a) (i)
(b) State one other factor that can affect the rate of this chemical reaction
(c) State the effect of a catalyst on the activation energy.

B7 Below is a skeleton diagram of the Periodic Table.

(a) Give the name of the region marked X on the Periodic Table above.

(b) Give any two general properties of elements found in the region marked X.

(c) State any two uses of the elements found in the region marked X.

Page 31 of 103 WhatsApp: 0954482350 / 0965038377 / 0974794056 OR EMAIL: laurentmsnd@gmail.com


LAURENT MUSONDA Call / Text: 0954482350
SUCCESS IN SCIENCE P2, 2015 – 2023 G.C.E DON’T POST / SHARE IT ON SOCIAL MEDIA

B8 Generally some metals easily corrode when exposed to certain conditions.


(a) Explain what is meant by corrosion.
…………………………………………………………………………… [1]
(b) Which of the metals zinc, potassium or copper corrodes fastest when exposed to
certain conditions?
……………………………………………………………………………. [1]
(c) Give two conditions that easily promote corrosion.
……………………………………………………………………………..[2]
(d) Give two methods of preventing corrosion of metals.
……………………………………………………………………………..[2]

B9 Carbon is asymmetrical and exists in many forms of allotropes.


(a) What are allotropes?

(b) Give two crystalline allotropes of carbon.

(c) Give one property and the use, based on the property, of one of the
allotropes you have named in (b) above.

Section C [20 marks]


Answer any two (2) questions from this section.
Write your answers in the separate answer booklet provided.

C1 The equations below show the thermal decomposition of the nitrates of three metals
represented by W, X and Y (not their real chemical symbols).
2WNO3  2W + 2NO2 + O2
2XNO3  2XNO2 + O2
2Y(NO3)2  2YO + 4NO2 + O2

(a) Which one of the three metals is the;


(i) Most reactive? Give a reason for your answer. [2]
(ii) Least reactive? Give a reason for your answer. [2]

(b) Which one of the three metals is likely to be sodium? [1]

(c) Which one of the three metals is


(i) more reactive than zinc? Give a reason for your answer. [2]
(ii) less reactive than zinc? Give a reason for your answer. [2]

(d) State the chemical test for oxygen. [1]

Page 32 of 103 WhatsApp: 0954482350 / 0965038377 / 0974794056 OR EMAIL: laurentmsnd@gmail.com


LAURENT MUSONDA Call / Text: 0954482350
SUCCESS IN SCIENCE P2, 2015 – 2023 G.C.E DON’T POST / SHARE IT ON SOCIAL MEDIA

C2 8.0g of impure sodium hydroxide solution reacted with exactly 40.0cm3 of 2.0mol/dm3
sulphuric acid.
(a) (i) Write the balanced chemical equation for the reaction.
Include state symbols. [3]
(ii) Construct a simplified net ionic equation for the reaction. [1]

(b) Calculate the number of moles for sulphuric acid that reacted with 8.0g of
impure sodium hydroxide. [2]

(c) Find the mass of pure sodium hydroxide from the impure solution that reacted with
40.0cm3 of sulphuric acid. [2]

(d) Determine the percentage purity of sodium hydroxide. [2]

C3 The series of processes below show the links to the formation of an ester known as ethyl
ethanoate.

(a) From the reaction scheme above, name processes K, L and M. [3]

(b) Construct the chemical equation describing process K. Include state symbols.

(c) State both the chemical name and common name for substance P. [2]

(d) State the general formula of the series where substance P belongs. [1]

(e) Give any two uses of ethyl ethanoate. [2]

Page 33 of 103 WhatsApp: 0954482350 / 0965038377 / 0974794056 OR EMAIL: laurentmsnd@gmail.com


LAURENT MUSONDA Call / Text: 0954482350
SUCCESS IN SCIENCE P2, 2015 – 2023 G.C.E DON’T POST / SHARE IT ON SOCIAL MEDIA

Centre Candidate
Candidate Name_________________________ Number Number

EXAMINATION COUNCIL OF ZAMBIA


Examination for General Certificate of Education Ordinary Level

Science 5124/2
Paper 2

2020

Time: 2 hours Marks: 85

SECTION A [20 marks]


Answer all the questions on the answer grid provided.

A1 A sample of a pure compound is heated until it is completely molten and the compound is
then allowed to cool until it is completely solid again.
The graph shows how the temperature of the compound changes with time.

Which of the following shows points when the compound exists in both liquid and solid
states?
A N to O
B O to P
C N to P
D P to Q

A2 The diagram below shows one of the methods used to separate mixtures.

Page 34 of 103 WhatsApp: 0954482350 / 0965038377 / 0974794056 OR EMAIL: laurentmsnd@gmail.com


LAURENT MUSONDA Call / Text: 0954482350
SUCCESS IN SCIENCE P2, 2015 – 2023 G.C.E DON’T POST / SHARE IT ON SOCIAL MEDIA

The above method is...


A chromatography. B crystallisation.
C decantation. D filtration.

A3 Which gas is not obtained on a large scale by fractional distillation?


A Ammonia B Argon
C Nitrogen D Oxygen

A4 A mixture of salt and iodine crystals was heated for 20 minutes as shown in the diagram
below.

What is R and S?
R S
A Residue Sublimate
B Sublimate Residue
C Fraction Distillate
D Distillate Fraction

A5 Graphite and diamond are allotropes of Carbon. Which of the statements below gives the
correct meaning of the term allotropy?
A The existence of two or more atoms having the same number of protons but
different number of neutrons.
B The existence of different forms of an element but in the same physical state.
C They are giant structures formed from a network of carbon atoms.
D Compounds having the same molecular formula but different structural formula.

Page 35 of 103 WhatsApp: 0954482350 / 0965038377 / 0974794056 OR EMAIL: laurentmsnd@gmail.com


LAURENT MUSONDA Call / Text: 0954482350
SUCCESS IN SCIENCE P2, 2015 – 2023 G.C.E DON’T POST / SHARE IT ON SOCIAL MEDIA

A6 The table below gives data about four substances. Which substance has particles in a
disorderly arrangement at room temperature?
Melting Point °C Boiling Point °C
A —114 —80
B 120 445
C 750 1 407
D 1 610

A7 At which temperature does a concentrated aqueous solution of sodium chloride begin to


boil?
A 98°C
B 99°C
C 100°C
D 104°C

A8 To determine the concentration of an acid, a learner titrated hydrochloric acid against


potassium hydroxide solution. She used 25.0cm3 of 0.2mol/dm3 potassium hydroxide. If she
started titrating from 0.7cm3 and reached her end point at 36.2cm3, what is the
concentration of the acid?
A 0.138 mol/dm3
B 0.141 mol/dm3
C 0.750 mol/dm3
D 1.141 mol/dm3

A9 The energy level profile shows how adding substance X to a reaction mixture changes the
reaction pathway.

Which change occurs when X is added to the reaction mixture?


A The rate of reaction decreases.
B The rate of reaction increases.
C The reaction becomes less exothermic.
D The reaction becomes more exothermic.

Page 36 of 103 WhatsApp: 0954482350 / 0965038377 / 0974794056 OR EMAIL: laurentmsnd@gmail.com


LAURENT MUSONDA Call / Text: 0954482350
SUCCESS IN SCIENCE P2, 2015 – 2023 G.C.E DON’T POST / SHARE IT ON SOCIAL MEDIA

A10 When solid potassium chloride is added to water, the temperature of the liquid goes down.
Which conclusion can be made from this observation?
A The potassium chloride molecules split into ions in water.
B The process is exothermic.
C The process is endothermic,
D Very little potassium chloride dissolves in water.

A11 Which ionic equation represents the neutralisation of sodium hydroxide solution with dilute
sulphuric acid?
A 2H+ (aq) + 2OH - (aq ) —> 2H2O (l)
B 2Na+ (aq) + SO4 2- (aq ) —> Na2SO4 (aq)
C 2Na+ (aq) + H2SO4 (aq ) —> Na2SO4 (aq) + 2H+
D +
2Na (aq) + H2SO4 (aq ) —> Na2SO4 (aq) + 2H2 (g)

Al2 Which of the following reagents could be used to distinguish between dilute nitric acid and
dilute hydrochloric acid?
A Aqueous barium chloride B Aqueous silver nitrate
C Aqueous sodium hydroxide D Copper (II) carbonate

A13 An aqueous solution of compound X reacts with sodium hydroxide solution to form a green
precipitate. When aluminium foil is added to the mixture and then warmed, a gas that turns
damp red litmus paper blue is given off. What is X?
A Ammonium nitrate B Copper (II) chloride
C Iron (II) nitrate D Iron (III) nitrate

A14 The positions of four elements are shown on the outline of part of the Periodic Table.
Element X has a high melting point and is a good conductor of electricity. It forms sulphates
XSO4 and X2(SO4)3. Which element is X?

A15 In the laboratory, an experiment was set up as shown in the diagram below.

Page 37 of 103 WhatsApp: 0954482350 / 0965038377 / 0974794056 OR EMAIL: laurentmsnd@gmail.com


LAURENT MUSONDA Call / Text: 0954482350
SUCCESS IN SCIENCE P2, 2015 – 2023 G.C.E DON’T POST / SHARE IT ON SOCIAL MEDIA

Which of the following is the correct equation for the reaction in the above experiment?
A Mg (s) + HCI (aq) —> MgCl2 (aq) + H2 (g)
B Mg (s) + 2HCI (aq) —> MgCl2 (aq) + 2H2 (g)
C Mg (s) + 2HCI (aq) —> MgCl2 (aq) + H2 (g)
D Mg (s) + HCI (aq) —> MgCl (aq) + H (g)

A16 The position of metal M in the reactivity series is shown below.


Na, M, Al, Zn, Fe, Pb, Cu, Ag
Which method will be used to extract M from its ore?
A Electrolysis of its molten oxide.
B Electrolysis of its aqueous sulphate.
C Reduction of its oxide by heating with coke.
D Reduction of its oxide by heating with hydrogen.

A17 A gas X was passed over a hot solid Y using the apparatus shown in the diagram below?

At the end of the experiment, the colour of Y had changed from black to pink and a white
precipitate formed in the limewater. What could be X and Y?
X Y
A Carbon dioxide Carbon
B Carbon dioxide Copper
C Carbon monoxide Copper
D Carbon monoxide Copper(II)oxide

A18 Three reactions involving ethanoic acid are represented by the letters X, Y and Z as shown in
the figure below.

Which of the reactions X, Y and Z involves oxidation?


A X only B X and Y
C Y only D Y and Z

Page 38 of 103 WhatsApp: 0954482350 / 0965038377 / 0974794056 OR EMAIL: laurentmsnd@gmail.com


LAURENT MUSONDA Call / Text: 0954482350
SUCCESS IN SCIENCE P2, 2015 – 2023 G.C.E DON’T POST / SHARE IT ON SOCIAL MEDIA

A19 The table shows the result of tests carried out on compound X.
Test Results
Bromine water added Decolourised
Sodium carbonate added Colourless gas evolved
Which formula represents compound X?

A20 Butane and methylpropane are isomers. Which formula is different for the two isomers?
A Empirical formula
B General formula
C Molecular formula
D Structural formula

Page 39 of 103 WhatsApp: 0954482350 / 0965038377 / 0974794056 OR EMAIL: laurentmsnd@gmail.com


LAURENT MUSONDA Call / Text: 0954482350
SUCCESS IN SCIENCE P2, 2015 – 2023 G.C.E DON’T POST / SHARE IT ON SOCIAL MEDIA

Section B [45 marks]


Answer all questions in this section.
Write your answers in the spaces provided in the question paper.
B1 The diagram below shows the set-up of apparatus used to investigate the products of a
burning candle.

(a) Why is the U-tube surrounded by ice cold water?


(b) Name the colourless liquid.
(c) State what happens to the lime water in the test tube.
(d) Name the elements found in candle wax.
(i) ……………………………………………………………
(ii) …………………………………………………………… [2]
(e) Explain how you can confirm the identity of the colourless liquid. [2]
[Total: 7 marks]
B2 The diagram below represents a paper chromatogram of three sugars K, M and N.

Page 40 of 103 WhatsApp: 0954482350 / 0965038377 / 0974794056 OR EMAIL: laurentmsnd@gmail.com


LAURENT MUSONDA Call / Text: 0954482350
SUCCESS IN SCIENCE P2, 2015 – 2023 G.C.E DON’T POST / SHARE IT ON SOCIAL MEDIA

(a) State the most soluble sugar. [1]


(b) On the Chromatogram above, indicate the solvent front. [1]
(c) The three sugars K, M and N are colourless; what should be done to the
chromatogram to make them visible. [2]
(d) Explain how chromatography can be used to identify false bank notes or forged
paper money. [2]
[Total: 6 marks]
B3 The grid below represents part of the Periodic Table. Study it and answer the questions
using the letters. The letters are not the actual symbols of the elements.

(a) An element V has atomic number 7. Indicate the position of V on the grid. [1]
(b) Explain why the atomic radius of E is bigger than that of R. [1]
(c) Which element has the highest tendency to gain electrons? [1]
(d) Select the most reactive metal, [1]
(e) Write the formula of a compound formed when B reacts with E. [1]

B4 (a) (i) Define concentration. [1]


(ii) State the SI unit for concentration. [1]
(iii) Calculate the mass of potassium hydroxide, KOH, that needs to be used to
prepare 500cm3 of 0.25M solution in water. [2]
(b) Construct a balanced chemical and net ionic equation, for the reaction between
potassium hydroxide and sulphuric acid. Include state symbols.
(i) Chemical equation: …………………………………………
(ii) Net ionic equation: …………………………………………. [3]
[Total: 7 marks]

B5 When excess magnesium powder was added to 100cm3 of 0.5M copper (II) sulphate
solution in a plastic cup wrapped in cotton wool, the temperature of the solution rose by
5°C.
(a) What type of reaction is this? [1]
(b) Suggest what was used to determine the rise in the temperature. [1]
(c) Describe what was observed when magnesium was added to the solution.
(i) ………………………………………………………………….
(ii) ………………………………………………………………… [2]
(d) Why is the plastic beaker wrapped in cotton wool during the experiment?
(e) How many moles of copper (II) sulphate are present in the 100cm3 solution? [2]
[Total: 7 marks]

Page 41 of 103 WhatsApp: 0954482350 / 0965038377 / 0974794056 OR EMAIL: laurentmsnd@gmail.com


LAURENT MUSONDA Call / Text: 0954482350
SUCCESS IN SCIENCE P2, 2015 – 2023 G.C.E DON’T POST / SHARE IT ON SOCIAL MEDIA

B6 Greenhouse gases pollute the air. Chlorofluorocarbons (CFCs) are greenhouse gases.
(a) (i) Name two other greenhouse gases found in the atmosphere
…………………………………..and………………………… [2]
(ii) State the respective origins of each of the gases named in part (i).
…………………………………………………………………………………………………………. [4]
(b) Describe how the presence of CFCs in the upper atmosphere increases the amount
of ultra violet light reaching the Earth's surface. [2]
[Total: 8 marks]
B7 The diagram below shows an arrangement of outermost electrons in a molecule of a
compound.

(a) (i) On the diagram indicate a double covalent bond by putting a circle
round it. [1]
(ii) Construct the chemical formula of the molecule above. [1]
(iii) Calculate the molecular mass of the molecule. [1]
(b) (i) To which homologous series of compounds does the molecule
belong to. [1]
(ii) State one chemical property of the molecule. [1]
[Total: 5 marks]
Section C [20 marks]
Answer any two (2) questions from this section. Write your answers in the separate answer booklet
provided.
C1 Clean dry air contains about 79% by volume of nitrogen.
(a) Name the gases which make up the remaining 21% by volume of clean air [2]
(b) A sample of polluted air was found to contain both nitrogen dioxide and sulphur
dioxide.
(i) State a major source of each of the pollutants. [2]
(ii) Describe a test which could be carried out to confirm the presence
of sulphur dioxide in the polluted air. [2]

Page 42 of 103 WhatsApp: 0954482350 / 0965038377 / 0974794056 OR EMAIL: laurentmsnd@gmail.com


LAURENT MUSONDA Call / Text: 0954482350
SUCCESS IN SCIENCE P2, 2015 – 2023 G.C.E DON’T POST / SHARE IT ON SOCIAL MEDIA

(c) Respiration and rusting are chemical processes which involve one of the gases in air.
(i) Show clearly, by chemical equations, how this gas takes part in
each of these reactions.
(ii) One method of preventing rusting is sacrificial protection.
Describe how this method of rust prevention could be used on an oil pipeline
and explain how it works. [4]

C2 (a) Define an acid. [1]


(a) Study the following reaction scheme.

(i) Give the names and formula of substances K to O. [5]


(ii) Write the balanced chemical equation for the, formation of solution
L and liquid M. [2]
(iii) Write an ionic equation for the formation of the white precipitate N.
Include state symbols. [2]
[Total: 10 marks]
C3 The table below shows information on some substances.
Substance Conducts electricity
when in solid state Melting point °C Dissolves in water
Aluminium Yes 660 No
Sodium chloride No 808 Yes
Sulphur No 113 No
Tungsten Yes 3 377 No
Wax No 35 to 50 No
Use the information in the table to answer the following questions.

(a) (i) Name two metals and give a reason for your answer.
(ii) Identify a mixture and justify your answer. [5]
(b) Write a chemical formula of the substance that is a compound. [1]
(c) Describe how the movement and arrangement of particles in aluminium change as
the temperature rises from 659°C to 661°C. [2]
(d) Identify the substance with the strongest bonds and give a reason for your answer.
[Total: 10 marks]

Page 43 of 103 WhatsApp: 0954482350 / 0965038377 / 0974794056 OR EMAIL: laurentmsnd@gmail.com


LAURENT MUSONDA Call / Text: 0954482350
SUCCESS IN SCIENCE P2, 2015 – 2023 G.C.E DON’T POST / SHARE IT ON SOCIAL MEDIA

Centre Candidate
Number Number

Candidate Name_________________________

EXAMINATION COUNCIL OF ZAMBIA


Examination for General Certificate of Education Ordinary Level

Science 5124/2
Paper 2

2021

Time: 2 hours Marks: 85

SECTION A [20 marks]


Answer all the questions on the answer grid provided in this question paper

A1 Which of the following is not a laboratory rule?


A Do not write on the board
B Do not eat while in the laboratory
C Always put on closed shoes
D Always close gas taps

A2 A learner accidentally mixed ammonium chloride and common salt. What method would
you use to separate the mixture?
A Decantation
B Filtration
C Floatation
D Sublimation

A3 The results of a paper chromatography experiment are shown in the chromatogram Below.
X is an aqueous solution of a salt of a Group I element
Y is an aqueous solution of a salt of a transition element

Page 44 of 103 WhatsApp: 0954482350 / 0965038377 / 0974794056 OR EMAIL: laurentmsnd@gmail.com


LAURENT MUSONDA Call / Text: 0954482350
SUCCESS IN SCIENCE P2, 2015 – 2023 G.C.E DON’T POST / SHARE IT ON SOCIAL MEDIA

Which row is correct?


Large Rf value Requires a locating
agent
A Y X
B X Y
C X X
D Y Y

A4 The following diagram is a cooling curve for a gas.

What process occurs at stage V?


A Boiling
B Condensation
C Melting
D Sublimation

A5 From the table below, which one is an ion?


Particles in the substance
Protons Neutrons Electrons
A 12 18 12
B 12 17 12
C 16 23 18
D 20 24 20

A6 Magnesium oxide has a high melting point. It is used to line the inside of furnaces that
operate at high temperatures. This is because it …
A has strong forces between its molecules.
B is a simple molecular substance.
C has metallic bonds.
D is an ionic compound.

Page 45 of 103 WhatsApp: 0954482350 / 0965038377 / 0974794056 OR EMAIL: laurentmsnd@gmail.com


LAURENT MUSONDA Call / Text: 0954482350
SUCCESS IN SCIENCE P2, 2015 – 2023 G.C.E DON’T POST / SHARE IT ON SOCIAL MEDIA

A7 Naturally occurring element Y has four isotopes whose relative abundance is shown in the
table below.
Isotope %
abundance
204 1.48
206 23.60
207 22.60
208 52.32
Find the relative atomic mass of the element
A 208.26
B 207.24
C 206.25
D 204.22

A8 The chemical formula for ammonia gas is NH3, Calculate the number of atoms contained in
85g of ammonia gas.
A 3.0 x 1025 atoms
B 1.2 x 1025 atoms
C 3.0 x 1024 atoms
D 2.4 x 1024 atoms

A9 Sodium hydroxide was used to neutralise sulphuric acid as shown in the equation below.
2NaOH + H2SO4 -------> Na2SO4 + 2H2O
How much sodium hydroxide would be used to neutralise 490 tons of sulphuric acid?
A 600 tons
B 450 tons
C 200 tons
D 150 tons

A10 A learner placed 200g of calcium carbonate into a conical flask and added 500cm3 of a 1.5
molar dilute hydrochloric acid solution.

What would remain in the flask at the end of the chemical reaction?
A Calcium chloride and water only.
B Aqueous calcium hydroxide only.
C Calcium carbonate, aqueous calcium hydroxide and water
D Calcium carbonate, aqueous calcium chloride and water

Page 46 of 103 WhatsApp: 0954482350 / 0965038377 / 0974794056 OR EMAIL: laurentmsnd@gmail.com


LAURENT MUSONDA Call / Text: 0954482350
SUCCESS IN SCIENCE P2, 2015 – 2023 G.C.E DON’T POST / SHARE IT ON SOCIAL MEDIA

A11 Solution Q reacted with aqueous sodium hydroxide producing a gas which changed the
colour of red litmus paper.
Which of the following could have been solution Q?
A Ammonium sulphate
B Calcium sulphate
C Potassium sulphate
D Sodium sulphate

A12 Salts can be classified as acidic, basic or neutral. Which of the following is a pair of basic
salts only?
A KCl and Na2CO3
B NHCI and CuSO4
C Na3PO4 and Ca(CH3COO)2
D ZnCl2 and BaSO4

A13 A bee sting is acidic. Which household substance will neutralise a bee sting?
A Damp bicarbonate of soda
B Damp common salt
C Lemon juice
D Vinegar

A14 Choose a statement that is most likely to be true about the elements in Group VIII of the
Periodic Table? They.
A are equally reactive chemically.
B form oxides of similar formulae.
C occur uncombined in nature.
D become less metallic as atomic number increases.

A15 Which of the following statements is correct about alloys? They are..
A mixtures made up of metals and other substances.
B Compounds made up of metals and other substances.
C Metals that are covalently bonded.
D formed by heavy metals only.

A16 Choose a fertilizer that contains all the three main elements needed by plants for normal
growth.
A NH4NO3
B KNO3
C NH4K2PO4
D (CO)(NH2)2

Page 47 of 103 WhatsApp: 0954482350 / 0965038377 / 0974794056 OR EMAIL: laurentmsnd@gmail.com


LAURENT MUSONDA Call / Text: 0954482350
SUCCESS IN SCIENCE P2, 2015 – 2023 G.C.E DON’T POST / SHARE IT ON SOCIAL MEDIA

A17 A gas escaping from a pipe in a chemical plant is tested and found to be alkaline. What is
this gas?
A Ammonia
B Carbon dioxide
C Hydrogen
D Oxygen

A18 Some organic compounds are generally non-reactive due to lack of a specific site for
chemical attack. Which of the following is such a compound?

A19 Choose one organic compound that makes up 98% of natural gas.
A Butane
B Ethane
C Methane
D Propane

A20 Which of the following structures belong to the same homologous series of compounds?

A (i), (iii) and (iv)


B (i), (ii) and (iii)
C (ii) and (iii)
D (i) and (iii)

Page 48 of 103 WhatsApp: 0954482350 / 0965038377 / 0974794056 OR EMAIL: laurentmsnd@gmail.com


LAURENT MUSONDA Call / Text: 0954482350
SUCCESS IN SCIENCE P2, 2015 – 2023 G.C.E DON’T POST / SHARE IT ON SOCIAL MEDIA

Section B [45 marks]


Answer all questions in this section.
Write your answers in the spaces provided on the question paper.

B1 Laboratory safety rules and regulatons are important to keep the laboratory safe.
(a) State the immediate action that a learner should take if the following happened
while carrying out an experiment:

(i) Chemicals got into contact with his or her clothes or skin or mouth.

(ii) She or he burnt or cut herself or himself.

(b) The following diagram shows a Bunsen burner, the most commonly used heating
apparatus in the chemistry laboratory.

Give one reason why the air hole should be fully open when the Bunsen burner is in
use.

(c) One of the gases used in a Bunsen burner is methane.

(i) State all products of combustion of this gas in the Bunsen burner when the
air-hole is fully open.
………………………………………………………………………………….
……………………………………………………………………………..[1]

(ii) One of the products of complete combustion of methane is a pollutant.


Name the product and state its effect on the environment.

Name …………………………………………………………………… [1]

Effect ……………………………………………………………………. [1]

[Total: 6 marks]

Page 49 of 103 WhatsApp: 0954482350 / 0965038377 / 0974794056 OR EMAIL: laurentmsnd@gmail.com


LAURENT MUSONDA Call / Text: 0954482350
SUCCESS IN SCIENCE P2, 2015 – 2023 G.C.E DON’T POST / SHARE IT ON SOCIAL MEDIA

B2 Figure B2.1 shows a separation technique used to separate miscible liquids.

Figure B2.1

(a) What is the name of this separation technique?


………………………………………………………………………………………….. [1]

(b) What is the purpose of the glass beads in the fractionating column.
…………………………………………………………………………………………………
……………………………………………………………………………………………. [1]
(c) Why is the Liebig condenser kept in the slanting position as shown in Figure B2.1
……………………………………………………………………………………………. [1]
(d) Apart from the mixture of miscible liquids, name two other mixtures which can be
separated using this separation technique.
………………………………………………………………………………………… [1]
………………………………………………………………………………………… [1]

Page 50 of 103 WhatsApp: 0954482350 / 0965038377 / 0974794056 OR EMAIL: laurentmsnd@gmail.com


LAURENT MUSONDA Call / Text: 0954482350
SUCCESS IN SCIENCE P2, 2015 – 2023 G.C.E DON’T POST / SHARE IT ON SOCIAL MEDIA

B3 2.0g of pure magnesium ribbon reacted with exactly 100cm3 of 2.0 mol/dm3 hydrochloric
acid.

(a) Construct a balanced chemical equation for the reaction. Include state symbols.
…………………………………………………………………………………….. [3]

(b) Determine the limiting reagent for the reaction.

[2]

(c) Calculate the volume of the gas that was evolved at r.t.p.
Volume ………………………………… [2]

[Total: 7 marks]

B4 (a) Table B4 contains atomic mass units (a.m.u) and percentage


(%) abundances for element Q.

Element a.m.u % abundance


Q 107 52
109 48
Table B4

Determine the relative atomic mass of element Q.

Relative atomic mass …………………………….. [2]

(b) Describe a radioisotope.


……………………………………………………………………………………………. [1]

(c) State one use of

(i) Carbon-14 isotope


………………………………………………………………………………. [1]

(ii) lodine-131 isotope.


………………………………………………………………………………. [1]
[Total: 5 marks]

Page 51 of 103 WhatsApp: 0954482350 / 0965038377 / 0974794056 OR EMAIL: laurentmsnd@gmail.com


LAURENT MUSONDA Call / Text: 0954482350
SUCCESS IN SCIENCE P2, 2015 – 2023 G.C.E DON’T POST / SHARE IT ON SOCIAL MEDIA

B5 Fluorine gas is bubbled through a solution of potassium bromide as shown in the diagram
below.

(a) (i) Describe what would be observed in the test tube.


……………………………………………………………………………… [1]

(ii) Write a balanced chemical equation for the reaction that occurs in the test
tube.
…………………………………………………………………………….. [2]

(b) What would be observed in the test tube if iodine gas was bubbled through the
potassium bromide solution? Give a reason for your answer.
Observation ……………………………………………………………………… [1]
Reason ……………………………………………………………………………… [1]
[Total: 5 marks]

B6 (a) Describe the rate of a chemical reaction.


………………………………………………………………………………………. [1]

(b) State a reason for each observation below.

(i) Hydrogen peroxide decomposes much faster in the presence of the enzyme
catalase.
………………………………………………………………………………. [2]

(ii) The reaction between manganese carbonate and dilute hydrochloric acid
speeds up when some concentrated hydrochloric acid is added.
……………………………………………………………………………… [2]

(iii) Powdered magnesium is used in fireworks rather than magnesium ribbon.

……………………………………………………………………………… [2]
[Total: 7 marks]

Page 52 of 103 WhatsApp: 0954482350 / 0965038377 / 0974794056 OR EMAIL: laurentmsnd@gmail.com


LAURENT MUSONDA Call / Text: 0954482350
SUCCESS IN SCIENCE P2, 2015 – 2023 G.C.E DON’T POST / SHARE IT ON SOCIAL MEDIA

B7 During the extraction of zinc, the ore is first roasted in air as illustrated by the equation
below

ZnS + O2 ------------> ZnO + SO2

(a) Give the common name of the ore in the equation above.
……………………………………………………………………………………….. [1]

(b) Name the substance which can be used to reduce zinc oxide, ZnO.
……………………………………………………………………………………… [1]
(c) Suggest a reason why sulphur dioxide should not be allowed to escape into the
atmosphere.
……………………………………………………………………………………………..
…………………………………………………………………………………………. [2]

(d) State one use of zinc.


…………………………………………………………………………………………. [1]
[Total: 5 marks]

B8 The structure displayed below is a polymer that was formed from monomer Y.

(a) Name the polymer.


……………………………………………………………………………………… [1]

(b) (i) Name the monomer Y.


……………………………………………………………………………[1]

(ii) Construct the displayed structure of monomer Y.


[1]

(c) Monomer Y was reacted with steam under Suitable conditions and an organic
compound Z was produced.

(i) To which homologous series does compound Z belong?


…………………………………………………………………………….. [1]

(ii) Write the general formula of the homologous series stated in (c)(i).
[Total: 5 marks]

Page 53 of 103 WhatsApp: 0954482350 / 0965038377 / 0974794056 OR EMAIL: laurentmsnd@gmail.com


LAURENT MUSONDA Call / Text: 0954482350
SUCCESS IN SCIENCE P2, 2015 – 2023 G.C.E DON’T POST / SHARE IT ON SOCIAL MEDIA

SECTIONC [20 marks]


Answer any two questions from this Section in the separate Answer Booklet provided.
C1 Lime is an important compound in industry.
(a) (i) State the chemical name of lime. [1]
(ii) Describe how lime is obtained from limestone on a large scale include an
equation in your answer. [2]
(b) (i) Give two uses of lime. [2]
(ii) What environmental problem is likely to arise or be caused by large scale
production of lime? Suggest a possible solution to the problem. [5]

[Total: 10 Marks]

C2 Indicators are pigments that can be extracted from flowers, fruits or roots and are used in
acid-base titrations.
(a) Describe how you can extract dyes or pigments from flowers. [4]
(b) (i) Name one salt that can be prepared by titration in the [1]
Laboratory and give a reason why titration is suitable for preparing the salt.
(ii) Describe an experiment you would carry out using the indicator solution
extracted as described in (a) to show that it is an effective indicator. [5]

[Total: 10 Marks]

C3 A hydrocarbon V which contains only 3 carbon atoms was bubbled through bromine
solution and there was a rapid decolourisation of the solution.

(a) (i) Identify the hydrocarbon V [1]


(ii) Draw the structure of the hydrocarbon V [1]
(iii) To which homologous series does the hydrocarbon V belong? [1]
(iv) Write the general formula of the homologous series where the Hydrocarbon
V belong. [1]

(b) (i) Explain the danger of burning the hydrocarbon V in limited Oxygen. [3]

(ii) Write the balanced chemical equation tor the reaction in (b)(i) above. [2]

(c) Under suitable conditions, the hydrocarbon V was polymerised and the polymer W
was produced. Display the structure of polymer W.

[Total: 10 Marks]

Page 54 of 103 WhatsApp: 0954482350 / 0965038377 / 0974794056 OR EMAIL: laurentmsnd@gmail.com


LAURENT MUSONDA Call / Text: 0954482350
SUCCESS IN SCIENCE P2, 2015 – 2023 G.C.E DON’T POST / SHARE IT ON SOCIAL MEDIA

Centre Candidate
Number Number

Candidate Name_________________________

EXAMINATION COUNCIL OF ZAMBIA


Examination for General Certificate of Education Ordinary Level

Science 5124/2
Paper 2

Wednesday 9 AUGUST 2022

Time: 2 hours Marks: 85

SECTION A [20 marks]


Answer all the questions on the answer grid provided in this question paper

A1 The branch of Chemistry that deals with the identification of composition of substances is...
A analytical chemistry.
B inorganic chemistry.
C organic chemistry.
D physical chemistry.

A2 The following diagrams show the arrangement of particles in a substance under normal
atmospheric pressure but at two different temperatures.

Which substance could the diagrams represent?


Substance Melting point/oC Boiling point/°C
A -150 -138
B -135 -120
C -120 -88
D -115 -92

Page 55 of 103 WhatsApp: 0954482350 / 0965038377 / 0974794056 OR EMAIL: laurentmsnd@gmail.com


LAURENT MUSONDA Call / Text: 0954482350
SUCCESS IN SCIENCE P2, 2015 – 2023 G.C.E DON’T POST / SHARE IT ON SOCIAL MEDIA

A3 The following diagram shows part of a laboratory thermometer.

What is the temperature shown in degrees Celsius?


A 32.0°C
B 30.2°C
C 28.0°C
D 20.8°C

A4 The following table lists the data obtained in a titration experiment. Which reading should
not be included to compute the average volume of the acid used in the titration?
Trial A B C D
Volume of acid/cm 3 12.9 13.0 12.8 11.9

A5 The following diagram shows a chromatogram obtained when sample V was analysed
together with four other known dyes W, X, Y, Z. Dye X was known to be poisonous.

Which of the following dyes are safe for use?


A V and W
B W only
C W and Z
D Z only

Page 56 of 103 WhatsApp: 0954482350 / 0965038377 / 0974794056 OR EMAIL: laurentmsnd@gmail.com


LAURENT MUSONDA Call / Text: 0954482350
SUCCESS IN SCIENCE P2, 2015 – 2023 G.C.E DON’T POST / SHARE IT ON SOCIAL MEDIA

A6 The following diagram shows the structure of an atom.

What is the proton number and nucleon number of the atom?


Proton Nucleon number
A 3 4
B 3 7
C 6 4
D 6 7

A7 Lithium occurs as a mixture of two stable isotopes: 63Li (7.5%) and 73Li (92.5%).
The relative atomic mass of lithium is …
A 6.2.
B 6.5.
C 6.9.
D 7.2.

A8 Magnesium forms Mg2+ (ions). The number of electrons in a magnesium ion is


A 6
B 10.
C 12.
D 14.

A9 Which elements exist as mono atomic molecules at room temperature and pressure?
A Helium, argon and neon.
B Fluorine, neon and argon.
C Chlorine, oxygen and helium.
D Hydrogen, helium and neon.

A10 Deduce the formula of lead (II) phosphate?


A PbPO4
B Pb2PO4
C Pb3(PO4)2
D Pb2(PO4)3

Page 57 of 103 WhatsApp: 0954482350 / 0965038377 / 0974794056 OR EMAIL: laurentmsnd@gmail.com


LAURENT MUSONDA Call / Text: 0954482350
SUCCESS IN SCIENCE P2, 2015 – 2023 G.C.E DON’T POST / SHARE IT ON SOCIAL MEDIA

A11 Which one of the following substances dissolves in water to form a solution with a pH of
less than 77
A Ammonia
B Magnesium sulphate
C Sodium chloride
D Sulphur dioxide

A12 Dilute sulphuric acid reacts with both magnesium oxide and magnesium carbonate. How
are these two reactions alike?
A Water is produced
B Hydrogen is produced
C Carbon dioxide is produced
D A white precipitate is formed

A13 Crystals of sodium carbonate decahydrate (Na2CO3.10H2O) are efflorescent. When these
crystals are exposed to air, the crystals ...
A gain mass and become liquid.
B gain mass and remain solid.
C lose mass and remain solid.
D lose mass, change to liquid and evolve bubbles of gas.

A14 One mole of nitrogen and one mole of ammonia gases have ...
A equal number of atoms.
B equal number of electrons.
C equal number of molecules.
D the same masses.

A15 A solution of calcium chloride contains 11.1g of the salt in 250cm3 of the solution. What is
the concentration of the solution in mol/dm3?
A 0.1
B 0.2
C 0.4
D 1.25

Page 58 of 103 WhatsApp: 0954482350 / 0965038377 / 0974794056 OR EMAIL: laurentmsnd@gmail.com


LAURENT MUSONDA Call / Text: 0954482350
SUCCESS IN SCIENCE P2, 2015 – 2023 G.C.E DON’T POST / SHARE IT ON SOCIAL MEDIA

A16 When an excess of magnesium powder was reacted with dilute hydrochloric acid, the
reaction was fast at first but gradually became slower and finally stopped.
Which of the following best explains why this happened? The…
A hydrochloric acid was used up.
B magnesium was used up.
C magnesium particles became coated with the hydrogen gas.
D temperature of the mixture decreased.

A17 What could be the chemical formula of the nitrate of an element Q of Group III of the
Periodic Table?
A QNO3
B Q3NO3
C Q(NO3)2
D Q(NO3)3

A18 Some metal ores are roasted in air before they are reduced in the blast furnace. What is
the main air pollution caused by this process?
A Carbon dioxide is produced.
B Carbon monoxide is produced.
C Lead fumes are produced.
D Sulphur dioxide is produced.

A19 The following structures represent different types of soap.

Which statement about soap is correct? Soap is .


A base.
B an ester
C a polymer.
D a salt.

Page 59 of 103 WhatsApp: 0954482350 / 0965038377 / 0974794056 OR EMAIL: laurentmsnd@gmail.com


LAURENT MUSONDA Call / Text: 0954482350
SUCCESS IN SCIENCE P2, 2015 – 2023 G.C.E DON’T POST / SHARE IT ON SOCIAL MEDIA

A20 What would be the structure of the polymer formed when the following monomers
undergo Polymerisation?

Section B [45 marks]


Answer all questions in this section.
Write your answers in the spaces provided on the question paper.

B1 The following experiment was set up to investigate factors that affect the rate
of diffusion in gases. During the experiment, a cloud of white smoke was seen
as the ammonia gas (NH3) and hydrogen chloride gas (HC) reacted.

Page 60 of 103 WhatsApp: 0954482350 / 0965038377 / 0974794056 OR EMAIL: laurentmsnd@gmail.com


LAURENT MUSONDA Call / Text: 0954482350
SUCCESS IN SCIENCE P2, 2015 – 2023 G.C.E DON’T POST / SHARE IT ON SOCIAL MEDIA

(a) Define the term diffusion.


……………………………………………………………………………………………………………………[1]
(b) Which of the particles for ammonia or hydrochloric acid will move faster?
……………………………………………………………………………………………………………………[1]
(c) (i) At which point A to D, along the tube T will the white cloud be seen?
…………………………………………………………………………………………………………[1]
(ii) Explain your answer in (c) (i) in terms of the movement of particles.
…………………………………………………………………………………………………………[1]
(d) State a
(i) factor that affects the rate of diffusion in the experiment above;
…………………………………………………………………………………………………………[1]
(ii) safety precaution you would take when carrying out the experiment.
…………………………………………………………………………………………………………[1]
[Total: 6 marks]

B2 The following are some common substances:


Air Water Limestone Sugar Common salt Brass Calcium petroleum

Choose from the above list a substance which is a


(a) compound containing three elements;
……………………………………………………………………………………………………………………[1]
(b) mixture of two elements;
……………………………………………………………………………………………………………………[1]
(c) mixture containing both elements and compounds;
……………………………………………………………………………………………………………………[1]
(d) a mixture of compounds.
……………………………………………………………………………………………………………………[1]
[Total: 4 marks]

B3 Atoms of J and K have 8 and 12 electrons respectively.


(a) Determine the formulae of the compound/molecule formed by the
combination of
(i) J and K; [1]
(ii) J and J. [1]
(b) In each of the cases shown in (a) (i) and (ii), name the type of chemical bond
formed. [2]
(c) Give two physical properties of the compound formed in (a)(i). [2]
[Total: 6 marks]

Page 61 of 103 WhatsApp: 0954482350 / 0965038377 / 0974794056 OR EMAIL: laurentmsnd@gmail.com


LAURENT MUSONDA Call / Text: 0954482350
SUCCESS IN SCIENCE P2, 2015 – 2023 G.C.E DON’T POST / SHARE IT ON SOCIAL MEDIA

B4 (a) What is the major characteristic feature of an acid?


……………………………………………………………………………………………………………………[1]
(b) Explain what is meant by each of the following terms:
(i) weak acid;
…………………………………………………………………………………………………………[1]
(ii) strong acid.
………….………………………………………………………………………………………………[1]
(c) Write equations to show the ionisation of
(i) dilute sulphuric acid;
…………………………………………………………………………………………………………[1]
(ii) ethanoic acid.
…………………………………………………………………………………………………………[1]
[Total: 5 marks]

B5 Excess of zinc metal was added to 200cm3 of 2M hydrochloric acid.


(a) Write a balanced chemical equation for the reaction.
……………………………………………………………………………………………………………………[2]

(b) Calculate the


(i) number of moles of hydrogen gas produced in the reaction;
[2]
(ii) volume of hydrogen gas produced at r.tp.
[1]
(c) What mass of zinc was used in the reaction?
[2]
[Total: 7 marks]

B6 The following table shows elements coded with letters Q to Y which do not represent the
actual symbol of any of the elements in the Periodic Table. You will need to use these
letters in some of your answers.

(a) Which element shown


(i) forms ions with a single negative charge;
…………………………………………………………………………………………………………[1]

Page 62 of 103 WhatsApp: 0954482350 / 0965038377 / 0974794056 OR EMAIL: laurentmsnd@gmail.com


LAURENT MUSONDA Call / Text: 0954482350
SUCCESS IN SCIENCE P2, 2015 – 2023 G.C.E DON’T POST / SHARE IT ON SOCIAL MEDIA

(ii) has three electrons in its outermost shell?


…………………………………………………………………………………………………………[1]
(b) How many protons does an atom of element Y contain?
…………….………………………………………………………………………………………………………[1]
(c) State the element which is the most reactive
(i) metal;
…………………………………………………………………………………………………………[1]
(ii) non-metal.
…………………………………………………………………………………………………………[1]
(d) Deduce the formula of the compound formed between elements U and
……………………….……………………………………………………………………………………………[1]
[Total: 6 marks]

B7 The following reaction scheme shows what occurs during the extraction of zinc metal
from one of its ores, zinc blende.
roasted burn
Zinc blende solid 𝐋 + sulphur dioxide zinc + gas 𝐌
in air > in coke >

(a) Deduce the chemical formula of zinc blende.


……………………….……………………………………………………………………………………………[1]
(b) Name solid L and gas M.
……………………….……………………………………………………………………………………………[2]
(c) Construct the chemical equation for the reaction of solid L with coke.
……………………….……………………………………………………………………………………………[1]
(d) Sulphur dioxide gas is a pollutant. Explain its effect on plants.
……………………….……………………………………………………………………………………………[2]
[Total: 6 marks]
B8 The four hydrocarbons given are members of a homologous series.
CH4 C4H10 C6H14 C10H22
(a) Explain the term 'hydrocarbon'.
……………………….……………………………………………………………………………………………[1]
(b) Name the
(i) homologous series to which the above hydrocarbons belong;
…………….……………………………………………………………………………………………[1]
(ii) compound with molecular formula C4H10 [1]
(c) Draw the full structural formula of the hydrocarbon which has four carbon atoms.
[1]
(d) Which of the given hydrocarbon has the highest boiling point? [1]

Page 63 of 103 WhatsApp: 0954482350 / 0965038377 / 0974794056 OR EMAIL: laurentmsnd@gmail.com


LAURENT MUSONDA Call / Text: 0954482350
SUCCESS IN SCIENCE P2, 2015 – 2023 G.C.E DON’T POST / SHARE IT ON SOCIAL MEDIA

SECTION C [20 marks]


Answer any two questions from this section in the separate Answer Booklet provided.
C1 The following scheme shows some reactions of a dilute acid A.

(a) Name and give the formulae of


(i) dilute acid A; [2]
(ii) substances B and C. [4]

(b) Construct
(i) balanced chemical equation for the reaction between substance B and
dilute acid A. [2]
(ii) an ionic equation for the reaction in (b) (i). [1]

(c) State the type of reaction that occurs in (b) (i). [1]
[Total: 10 marks]

C2 Calcium hydroxide or slaked lime is an industrial alkali.


(a) Explain
(i) the meaning of the term alkali; [1]
(ii) how calcium hydroxide is produced starting with limestone. [3]

(b) Give two large scale uses of calcium hydroxide. [2]

(c) A weak solution of calcium hydroxide is called limewater and it is used to test for
carbon dioxide gas.
(i) Construct a chemical equation for the reaction. Include state symbols. [2]
(ii) Give the chemical name of the salt produced during the reaction. [1]
(iii) Describe what is observed during the reaction. [1]
[Total: 10 marks]

Page 64 of 103 WhatsApp: 0954482350 / 0965038377 / 0974794056 OR EMAIL: laurentmsnd@gmail.com


LAURENT MUSONDA Call / Text: 0954482350
SUCCESS IN SCIENCE P2, 2015 – 2023 G.C.E DON’T POST / SHARE IT ON SOCIAL MEDIA

C3 Propene is one of the many important hydrocarbons obtained from crude oil. LiKe
propane, it is made up of molecules which contain three carbon atoms.

(a) (i) Draw a structural formula for propene. [1]


(i) How does propene differ from propane in terms of bonds? [1]
(iii) To which group of hydrocarbons does propene belong? [1]
(iv) Give the molecular formula for propane and propene respectively.[2]

(b) (i) Explain why propene reacts immediately with bromine water whereas
propane does not. [2]
(ii) Describe what would be observed during the reaction. [1]

(c) Propene is obtained by breaking down longer chains of hydrocarbons.


What is this process called? [1]

(d) Propene is a monomer for making an important plastic. State the name of the
polymer it forms. [1]
[Total: 10 marks]

Page 65 of 103 WhatsApp: 0954482350 / 0965038377 / 0974794056 OR EMAIL: laurentmsnd@gmail.com


LAURENT MUSONDA Call / Text: 0954482350
SUCCESS IN SCIENCE P2, 2015 – 2023 G.C.E DON’T POST / SHARE IT ON SOCIAL MEDIA

EXAMINATIONS COUNCIL OF ZAMBIA


Examination for General Certificate of Education Ordinary Level

Science 5124/2
Paper 2

Wednesday 9 AUGUST 2023


Time: 2 hours Marks: 85

SECTION A [20 marks]

A1 Which of the following is not a branch of Chemistry?


A Biochemistry
B Organic Chemistry
C Physical Chemistry
D Thermodynamics

A2 Which of the following statements distinguishes between an atom and an ion? An atom …
A has different numbers of protons and electrons while an ion has equal number of
protons and electrons.
B has no protons while an ion has protons.
C is electrically charged while an ion is electrically neutral.
D is electrically neutral while an ion has an overall electrical charge.

A3 Which of the following pieces of apparatus can be used to measure accurately a fixed
volume of a liquid?
A Burette
B Measuring cylinder
C Pipette
D Separating funnel

A4 Identify a separation technique which can be used to separate a mixture of water and butanol.
A Decantation B Evaporation
C Fractional distillation D Separating funnel

Page 66 of 103 WhatsApp: 0954482350 / 0965038377 / 0974794056 OR EMAIL: laurentmsnd@gmail.com


LAURENT MUSONDA Call / Text: 0954482350
SUCCESS IN SCIENCE P2, 2015 – 2023 G.C.E DON’T POST / SHARE IT ON SOCIAL MEDIA

A5 A compound is a …
A chemical combination of two or more elements.
B combination of two or more mixtures.
C mixture of two or more elements.
D physical combination of two or more elements,

A6 Which pair of elements form a compound by sharing electrons?


A Argon and oxygen
B Nitrogen and oxygen
C Potassium and iodine
D Sodium and chlorine

A7 An element X forms an ion 60 3+


27X . What is the number of electrons, neutrons and protons
in this ion?
Electrons Neutrons Protons
A 27 13 27
B 27 33 30
C 24 33 27
D 24 60 24

A8 Which of the following is not a characteristic of an acid? An acid …


A has a sour taste.
B produces hydroxide ion (OH) when dissolved in water.
C reacts with a metal carbonate to produce a salt, water and carbon dioxide gas,
D turns a blue litmus paper red.

A9 Which statement explains why salts do not conduct electricity in a solid state of matter?
A The delocalised electrons in salts are not free and not mobile in solid state,
B The ions in salts are not free and not mobile in solid state.
C They have free and mobile ions in them which help conduct electricity in solid
states.
D They have free and mobile delocalised electrons in solid state.

Page 67 of 103 WhatsApp: 0954482350 / 0965038377 / 0974794056 OR EMAIL: laurentmsnd@gmail.com


LAURENT MUSONDA Call / Text: 0954482350
SUCCESS IN SCIENCE P2, 2015 – 2023 G.C.E DON’T POST / SHARE IT ON SOCIAL MEDIA

A10 When strongly heated, calcium carbonate decomposes according to the chemical equation
shown:

Given that 500g of calcium carbonate (CaCO3) were thermally decomposed, calculate the
Volume of carbon dioxide gas which was produced at r.t.p.
A 120dm3 B 90dm3
C 60dm3 D 30dm3

A11 Why should water never be added to an acid when diluting a concentrated acid?
A The process of diluting acids is endothermic and concentrated acids have low
affinity for water.
B The process of diluting acids is exothermic and concentrated acids have a high
affinity for water
C Water and acids are immiscible
D Water and acids are miscible.

A12 The following graphs summarise the reaction between powdered calcium carbonate a
dilute hydrochloric acid of different concentrations and show the volume of gas produced.

Which graph shows the slowest chemical reaction?


A 1
B 2
C 3
D 4

A13 Element X has valency electrons equal to six (6). In which Group of the Periodic Table does
element X belong?
A 6
B 4
C 3
D 2

Page 68 of 103 WhatsApp: 0954482350 / 0965038377 / 0974794056 OR EMAIL: laurentmsnd@gmail.com


LAURENT MUSONDA Call / Text: 0954482350
SUCCESS IN SCIENCE P2, 2015 – 2023 G.C.E DON’T POST / SHARE IT ON SOCIAL MEDIA

A14 How are elements on the Periodic Table arranged? They are arranged according to their …
A boiling points.
B mass numbers.
C melting points
D proton numbers.

A15 Which of the following explains why non-metals are poor conductors of heat and
electricity?
A The electrons in them are localised and not mobile.
B The ions in them are free and mobile.
C The molecules in them are delocalised and are free and mobile.
D They have delocalised electrons which are free and mobile.

A16 Which of the following is not a property of a typical metal?


A Brittleness
B Conducting electricity
C Ductility
D Sonorosity

A17 Hydrogen can be manufactured from methane according to the following reaction scheme.

What are the identities of substances M, N and P?


M N P
A Nickel Oxygen Oxides of carbon
B Steam Oxygen Oxides of carbon
C Steam Nickel Oxides of carbon
D Oxygen Steam Oxides of carbon

A18 Which of the following is true about chlorine, bromine and iodine? They...
A are coloured.
B are good conductors of electricity.
C are non-poisonous.
D have no smell when in the gas phase.

Page 69 of 103 WhatsApp: 0954482350 / 0965038377 / 0974794056 OR EMAIL: laurentmsnd@gmail.com


LAURENT MUSONDA Call / Text: 0954482350
SUCCESS IN SCIENCE P2, 2015 – 2023 G.C.E DON’T POST / SHARE IT ON SOCIAL MEDIA

A19 Which of the following are the products formed in a chemical reaction between an alcohol
and a carboxylic acid?
A An ester and water only.
B A salt and hydrogen gas.
C A salt, carbon dioxide gas and water
D A salt only.

A20 The following structure shows a common polymer which has 3 monomer units.

Name the monomer for this polymer.


A Carbon
B Cellulose
C Glucose
D Oxygen

Section B [45 marks]


Answer all questions in this section. Write your answers in the spaces provided in this question
paper
B1 (a) (i) Define the term 'matter'
…………………………………………………………………………………………………………….. [1]

(ii) Give the basic unit of matter contained in carbon dioxide.


…………………………………………………………………………………………………………….. [1]

(b) State two challenges that have been brought by activities done in chemical industries.
(i) …………………………………………………………………………………………………………….. [1]
(ii) …………………………………………………………………………………………………………….. [1]

(c) Explain why it is not advisable for learners to enter the laboratory in the absence of
a teacher.
…………..………………………………………………………………………………………………………….. [1]
[Total: 5 marks]
B2 Use the knowledge of the Kinetic Theory of matter to answer the questions that follow.
(a) Explain why an inflated balloon which is placed in a refrigerator slowly starts to
shrink and reduce in volume.
………………………………………………………………………………………………………………………….. [1]

Page 70 of 103 WhatsApp: 0954482350 / 0965038377 / 0974794056 OR EMAIL: laurentmsnd@gmail.com


LAURENT MUSONDA Call / Text: 0954482350
SUCCESS IN SCIENCE P2, 2015 – 2023 G.C.E DON’T POST / SHARE IT ON SOCIAL MEDIA

(b) With reference to energy changes, what type of process takes place in (a)?
……………..…………………………………………………………………………………………………………….. [1]

(c) Explain why wet clothes dry faster on a warmer day than on a colder day.
………………………………………………………………………………………………………………………….. [1]

[Total: 5 marks]

B3 The experimental set up that follows shows a mixture of sand and iodine Crystals in an
evaporating dish.

(a) Describe how the mixture of sand and iodine crystals can appropriately be
separated.
……………..…………………………………………………………………………………………………………….. [2]

(b) Explain why bottled mineral water is not pure water.


……………..…………………………………………………………………………………………………………….. [2]

(c) Give one criterion for purity of a substance,


……………..…………………………………………………………………………………………………………….. [1]
[Total: 5 marks]
B4 (a) Figure 4.1 shows an electronic structure for element Q.

(i) Name element Q.


……..……………………………………………………………………………………………………….. [1]

Page 71 of 103 WhatsApp: 0954482350 / 0965038377 / 0974794056 OR EMAIL: laurentmsnd@gmail.com


LAURENT MUSONDA Call / Text: 0954482350
SUCCESS IN SCIENCE P2, 2015 – 2023 G.C.E DON’T POST / SHARE IT ON SOCIAL MEDIA

(ii) State the period of the Periodic Table where element Q belongs.
Explain your answer.
……..……………………………………………………………………………………………………….. [2]

(iii) Explain how an atom of element Q acquires chemical stability.


……..……………………………………………………………………………………………………….. [1]

(b) Explain what you understand by the term 'nucleon number.


……………..………………………………………………………………………………………………………….. [1]
[Total: 5 marks]
B5 (a) Distinguish between an acid and a base.
……………..………………………………………………………………………………………………………….. [2]

(b) Explain what you understand by the term 'salt.


……………..………………………………………………………………………………………………………….. [2]
[Total: 4 marks]
B6 (a) Define the term 'mole'.
……………..………………………………………………………………………………………………………….. [1]

(b) The concentration of sodium hydroxide solution is 0.25mol/dm3. Calculate the


number of moles of 0.25mol/dm3 of sodium hydroxide solution contained in
250cm3 of this solution.

[2]
(c) Iron metal is extracted from an iron ore called haematite in a blast furnace during a
reduction reaction using carbon monoxide as a reducing agent as shown in the
following chemical equation:
Fe2O3 (s) + 3CO(g) ---------> 2Fe(l) + 3CO2(g).
Calculate the mass of carton monoxide gas required to reduce 3 200kg of iron (III)
oxide (Fe2O3) in the blast furnace.

[2]
[Total: 5 marks]

Page 72 of 103 WhatsApp: 0954482350 / 0965038377 / 0974794056 OR EMAIL: laurentmsnd@gmail.com


LAURENT MUSONDA Call / Text: 0954482350
SUCCESS IN SCIENCE P2, 2015 – 2023 G.C.E DON’T POST / SHARE IT ON SOCIAL MEDIA

B7 (a) Describe how the action of heat on the carbonate of a metal depends on the
position of the metal in the reactivity series.
……………..………………………………………………………………………………………………………….. [2]

(b) Transition metals and their compounds can be used as catalysts.


(i) Explain what is meant by the term 'catalyst.
..………………………………………………………………………………………………………….. [1]

(ii) Explain why metals are good conductors of electricity and heat.
..…………………………………………………………………………………………………………….. [2]
[Total: 5 marks]
B8 Greenhouse gases are given greenhouse factor values. The greenhouse effect of a gas
increase with the factor value. The following Table 8.1 gives information gases

Greenhouse gas Greenhouse Percentage of gas in


factor the atmosphere
Carbon dioxide (CO2) 1 0.036
Methane, (CH4) 30 0.0017
Dinitrogen oxide, (N2O) 160 3.0 x 10-4
Carbontrichlorofluoride 21, 000 2.8 x 10-8
(CCl3F)
Table 8.1

(a) ldentify the gas which has the greatest effect on the environment. Give a reason for
your answer.
……………..………………………………………………………………………………………………………….. [2]

(b) Why is an increase in the percentage of methane more worrying than the same
percentage increase of carbon dioxide?
……………..………………………………………………………………………………………………………….. [2]

(c) What other environmental problem, besides its action as a greenhouse gas, is
caused by carbontrichlorofluoride, (CCl3F)?
……………..………………………………………………………………………………………………………….. [2]
[Total: 6 marks]

Page 73 of 103 WhatsApp: 0954482350 / 0965038377 / 0974794056 OR EMAIL: laurentmsnd@gmail.com


LAURENT MUSONDA Call / Text: 0954482350
SUCCESS IN SCIENCE P2, 2015 – 2023 G.C.E DON’T POST / SHARE IT ON SOCIAL MEDIA

B9 Alkanes from a homologous series of organic compounds which are also called paraffins.
(a) (i) State the general molecular formula to which the alkanes conform.
……..……………………………………………………………………………………………………….. [1]

(ii) Draw the displayed structural formula of pentane.

[1]
(b) Give two reasons why cracking of large hydrocarbons is Important.
(i) ……..…………………………………………………………………………………………………….. [1]
(ii) ……..……………………………………………………………………………………………………….. [1]

(c) Justify why alkanes are sometimes called paraffins.


……………..……………………………………………………………………………………………………….. [2]
[Total: 5 marks]

Section C [20 marks]

Answer any two questions from this section in the separate Answer Booklet provided.

C1 The scale in a kettle contains calcium carbonate and aminosulphonic acid is used to
remove this scale. This acid reacts with the scale and produces a gas,
(a) The apparatus In Figure C1.1 was used to collect the gas formed when excess acid
was added to small amounts of the scale.

Figure C1.1
(i) How could you tell that a gas is being produced? [1]
(ii) Name the gas produced. [1]
(iii) Describe a chemical test for the gas produced. [2]

Page 74 of 103 WhatsApp: 0954482350 / 0965038377 / 0974794056 OR EMAIL: laurentmsnd@gmail.com


LAURENT MUSONDA Call / Text: 0954482350
SUCCESS IN SCIENCE P2, 2015 – 2023 G.C.E DON’T POST / SHARE IT ON SOCIAL MEDIA

(b) A learner measured the total volume of gas produced over 120 seconds. His results
are shown in Table C1.1.
Time/s 0 5 20 35 55 70 90 120
Total volume/cm 3 0 20 40 50 60 75 65 65
Table C1.1

(i) Plot a graph of the results (volume against time) on the graph paper
provided. [3]
(ii) Circle the point which seems to be incorrect. [1]
(iii) Calculate the average rate of this reaction. [2]
[Total: 10 marks]
C2 Zinc blende is one of the ores of zinc metal.
(a) State the chemical name of zinc blende. [1]
(b) Describe how zinc metal is extracted from zinc blende. Include the necessary
balanced chemical equations. [6]
(c) State two uses of zinc metal. [2]
(d) Suggest one environmental problem that is a consequence of extraction of metals. [1]
[Total: 10 marks]

C3 Ethane and ethene are both organic compounds.


(a) Compare how they react, if at all with;
(i) oxygen gas, [2]
(ii) hydrogen gas. [2]
(iii) Write a chemical equation for any of the reactions in (i) or (ii). [2]

(b) Draw the displayed structural formula of


(i) ethane,
(ii) ethene [2]
(e) Describe a chemical test that can be carried out to distinguish ethane from ethene. [2]
[Total: 10 marks]

Page 75 of 103 WhatsApp: 0954482350 / 0965038377 / 0974794056 OR EMAIL: laurentmsnd@gmail.com


LAURENT MUSONDA Call / Text: 0954482350
SUCCESS IN SCIENCE P2, 2015 – 2023 G.C.E DON’T POST / SHARE IT ON SOCIAL MEDIA

ANSWERS FOR SCIENCE PAPER 3 (CHEMISTRY) – 2015


ANSWERS FOR SECTION A [45 MARKS]
1 (a) Evaporation
(b) Evaporation dish
(c) Iodine
(d) NaCl can be separated from water by physical reaction

2 (a) Nucleon number is the total number of protons and neutrons in an atom.
(b)
Number of particles Total number in one atom
Neutrons 12
Protons 11
Electrons 11

(c) Na+10: 2, 8
3 (a) A – Cracking
S – Hydration
(b) B – Ethene
(c) Cracking help to produce short – chain compounds with (C=C) double bond which
makes it to be very reactive.

4 (a) Combustion of fuels in car exhausts.


(b) Lead damage the bones, brain and the nervous system.
Lead cause children to have low intelligence, poorer memories and less muscular
coordination.
(c) Lead tetraethyl is added to petrol as an inhibitor to prevent knocking
(d) (i) Global warming is the rising of average world’s temperature due to heat
absorbed by carbon dioxide in the atmosphere.
(ii) Carbon dioxide
5 (a) Blue
(b) cause water turned into alkaline solution
(c) (i) Hydrogen
(ii) it put off a burning splint with pop sound
(d) 2K (s) + 2H2O (l)  2KOH (aq) + H2 (g)
6 (a) (i) because it will be added to sulphuric acid until the solution saturated.
(ii) Through filtration.
(b) m = (96 x 2) ÷ 24 = 8g

7 (a) Potassium iodine is colourless when bromine gas bubbled in it, it will turned to
shade brown because of potassium bromide which is formed.
(b) 2KI (aq) + Br2 (g)  2KBr (aq) + I2 (g)
(c) Displacement reaction
(d) Iodine is less reactive than bromine.
(e) 7 electrons in their outermost shell

Page 76 of 103 WhatsApp: 0954482350 / 0965038377 / 0974794056 OR EMAIL: laurentmsnd@gmail.com


LAURENT MUSONDA Call / Text: 0954482350
SUCCESS IN SCIENCE P2, 2015 – 2023 G.C.E DON’T POST / SHARE IT ON SOCIAL MEDIA

8 (a) Sodium, Aluminium, Zinc, Copper


(b) (i) Copper.
(ii) Copper pyrites.
(c) Because when aluminium exposes to air it reacts with oxygen in air forming a layer
called aluminium oxide which protect aluminium further to react with oxygen in air.
9 (a) Both jar M and jar N will be full of bromine gas.
(b) Because of diffusion of bromine gas from jar N to jar M.
ANSWERS FOR SECTION B [20 MARKS]
10 (a) copper (ii) oxide
(b) Heat sulphuric acid in a beaker. Add copper (ii) oxide while stirring until the solution
is saturated. Filter the saturated solution to remove undissolved copper (ii) oxide.
Put the filtrate on evaporating dish to evaporate water of crystallization.
(c) CuO(s) + H2SO4(aq)CuSO4 (aq) + H2O (i)
(d) m = (19.6 x 287) ÷ 98 = 57.4 g
(e) Water of Crystallisation is water molecules present in one molecule of salt.

11 (a) (i) Filtrate is a solution or liquid that has passed through a filter paper
during filtration.
(ii) Residue is substance that remained on a filter paper during the process of
filtration.
(b) By decantation; solid and liquid are let to settle and pour out liquid and solid
remain.
By filtration; liquid pass through a filter paper and solid remain as residue.
(c) (i) Fractional distillation has a fractionating column while Simple distillation
doesn’t have.
(ii) Separation of crude oil.
Separation of fermented liquor into ethanol and water.

12 (a) This is an acid which ionises partially in water.


(b) 2CH3CH2COOH + K2CO3  2CH3CH2COOK + CO2 + H2O
(c) Mass = 2g
(d) (i) COOH – Carboxylic group
OH – Alcohol group
(ii) Condensation polymerisation
(e) Throwing them in cities causes litter problem
burning them causes air pollution

ANSWERS FOR SCIENCE PAPER 1 - 2016 (CHEMISTRY PART)


1. B 6. A 11. B 16. A
2. B 7. A 12. D 17. B
3. C 8. D 13. A 18. A
4. A 9. D 14. C 19. D
5. B 10. B1 15. D 20. D

Page 77 of 103 WhatsApp: 0954482350 / 0965038377 / 0974794056 OR EMAIL: laurentmsnd@gmail.com


LAURENT MUSONDA Call / Text: 0954482350
SUCCESS IN SCIENCE P2, 2015 – 2023 G.C.E DON’T POST / SHARE IT ON SOCIAL MEDIA

ANSWERS FOR SCIENCE P3 (CHEMISTRY) – 12 JULY 2016


SECTION A [45 MARKS]
1 (a) (1) ions (2) atoms (3) molecules
(b) Spraying perfume at the corner of the classroom, the smell will fill the whole room.
Dropping potassium manganite (VII) into water and it will spread throughout the
water until it forms a uniform colour.
(c) State: Liquid state
Explanation: Particles are sliding over to each other.

2 (a) Condensation is the change of state of matter from solid to liquid. Melting
(b) The basic units of matter that exist in sodium chloride are molecules. Ions
(c) Isotopes are compounds with the same molecular formula but different structural
formulae. Isomers
(d) Calcium hydroxide is classified as an amphoteric substance because it dissolves in
sodium hydroxide solution. Aluminium oxide
(e) Zinc and chlorine bond covalently. Ionic bond
3 (a) Atom of element A Atom of element D

(b) (i) ionic bond


(ii)

(c) (1) It conduct electricity in molten or aqueous state.


(2) It has high melting and boiling point.

4 (a) Calcium hydroxide + Carbon dioxide ------> Calcium carbonate + water.


(b) (i) n = V ÷ Vm = 1.2 ÷ 24 = 0.05mol
m= n x mr = 0.05 x 44 = 2.2g
m = (74 x 12) ÷ 44 = 3.7g
(ii) m = ( 2.2 x 100 ) ÷ 44 = 5g
(iii) white precipitates will be seen

Page 78 of 103 WhatsApp: 0954482350 / 0965038377 / 0974794056 OR EMAIL: laurentmsnd@gmail.com


LAURENT MUSONDA Call / Text: 0954482350
SUCCESS IN SCIENCE P2, 2015 – 2023 G.C.E DON’T POST / SHARE IT ON SOCIAL MEDIA

5 (a)
Name of the Name of the Displayed full structural
homologous series member formula
Alkenes
Ethane

Ethanol
Alcohol

carboxylic acid Ethanoic acid

6 (a) Sodium (Na) (b) Potassium (K)


(c) Copper (Cu) (d) Argon (Ar)
7 (a) NaOH + HCl ---------> NaCl + H2O
(b) Type of reaction: Neutralization reaction
Ionic equation: OH- (aq) + H+ (aq) -----> H2O (l)
(c) The solution is heated to evaporate the water to saturation point. The solution is
cooled and crystals are formed. The pure crystals of sodium chloride are filtered,
washed and dried between two filter papers.

8 (a) Redox reaction


(b) (i) Substance oxidized: Magnesium
Reason: Oxygen has been added to magnesium to form magnesium oxide.
(ii) Substance oxidized: Carbon
Reason: Oxygen has been removed from carbon dioxide to form carbon.
(c) Carbon
(d) Carbon dioxide (CO2) is Acidic oxide
Magnesium oxide (MgO) is Basic oxide

SECTION B [20 MARKS]


9 (a) Cu2S (g) + O2 (g) ---------> Cu (s) + SO2 (g)
(b) Because it produces a poisonous gas called sulphur dioxide which chock the throat.
(c) Electrolysis
(d) (1) Copper is used in making electrical connecting wire because it is a good
conductor of electricity (have mobile electrons).
(2) Copper is used to make car radiator because copper is unreactive with
water and cannot form rust.
(e) (i) Native means unreactive
(ii) Brass / Bronze

Page 79 of 103 WhatsApp: 0954482350 / 0965038377 / 0974794056 OR EMAIL: laurentmsnd@gmail.com


LAURENT MUSONDA Call / Text: 0954482350
SUCCESS IN SCIENCE P2, 2015 – 2023 G.C.E DON’T POST / SHARE IT ON SOCIAL MEDIA

10 (a) (NH4)2SO4 (aq) + 2NaOH (aq) ------> Na2SO4 (aq) + 2H2O (l) + 2NH3 (g)
(b) You place a red litmus paper to the vapour and if it turns blue meaning ammonia is
produced.
(c) (i) Hydrogen gas and Nitrogen gas
(ii) 3H2 (g) + N2 (g) ---------> 2NH3 (g)
(iii) (1) Catalyst (iron)
(2) Temperature between 250 oC - 350 oC
(3) Pressure about 450 atm

11 (a) (1) Temperature about 300oC


(2) Catalyst known as Phosphoric acid (H3PO4)
(3) Pressure about 65 atmospheres
(b) Addition reaction under hydration reaction
(c) (i) no answer - error (No minor by product is form between alkene and steam)
(ii) no answer – error
(iii) m = 2.1 x 60 ÷ 42 = 3g
(d) CnH2n

ANSWERS FOR SCIENCE PAPER 2 – 1 AUGUST 2017


SECTION A [20 Marks]
1. B 5. D 9. D 13. D 17. C
2. D 6. D 10. B 14. D 18. B
3. A 7. C 11. D 15. B 19. B
4. A 8. D 12. C 16. A 20. A

SECTION B [45 Marks]


B1 (a) H2SO4 + Ca(OH)2  CaSO4 + 2H2O
(b) Mr of Ca(OH)2 = 40 + 32 + 2 = 74g/mol
(c) Mass of calcium sulphate = (15.5 x 74 ) ÷ 98 = 11.705 tonnes
(d) Hydroxide ion (OH-)

B2 (a) (i) Coke


(ii) Limestone
(b) Fe2O3 + 3CO  2Fe + 3CO2
(a) (i) Presence of water vapour;
(ii) Presence of Oxygen

B3 (a) Iron (b) Lithium (c) Sulphur (d) Mercury (e) Iron

B4 (a) Exhaust from car engine; power station/ factories


(b) (i) It causes acid rain.
(ii) Acid rain reacts with lime forming salt hence lime will be washed away from
the buildings.
(c) Carbon

Page 80 of 103 WhatsApp: 0954482350 / 0965038377 / 0974794056 OR EMAIL: laurentmsnd@gmail.com


LAURENT MUSONDA Call / Text: 0954482350
SUCCESS IN SCIENCE P2, 2015 – 2023 G.C.E DON’T POST / SHARE IT ON SOCIAL MEDIA

B5 (a) Precipitation (b) Displacement (c) Neutralisation (d) Combustion (d) Synthesis

B6 (a) Condensation (b) Melting (c) Sublimation (d) Dissolving (e) Chemical change
B7 (a) (i) Endothermic reaction – this is a reaction in which energy is absorbed
from the surrounding.
(ii) Exothermic reaction – this is reaction in which energy is released to the
surrounding.
(b) Endothermic reaction –e.g. Photosynthesis, melting of ice, absorption of light in
photography.
Exothermic reaction – Respiration, combustion (burning of papers), freezing of
liquid.
(c) Combustion produces carbon monoxide which causes air pollution.

B8 (a) Agriculture – farmers use fertilizers made from chemical elements such as
nitrogen, phosphorous to improve soil fertility.
Mining industry – chemistry is applied in the extraction of metals such as copper
from copper ores.
(b) (i) You should always wear shoes in the laboratory.
(ii) Do not use broken glassware

B9 (a) Y, Z, X, W
(b) Metal X
(c) By reducing it with carbon or carbon monoxide.
(d) ZCO3 (s)  ZO (s) + CO2 (g)
(e) Silver because it does not rust

SECTION C [20 marks]


C1 (a) (i) Polypropene
(ii) Name – propene. Structure –

(b) (i) Synthetic polymer


(ii) To make plastics
(c) Cassava is crushed and pressure cooked to release starch (glucose). Then it
will be mixed with water and yeast then allow it to react for few days in the
absence of air (oxygen) at the temperature of 37oC. The enzyme in yeast
(zymase) will help to convert starch to ethanol and carbon dioxide gas will be
produced.
(d) C2H5OH (l) + 3O2  2CO2 (g) + 3H2O (l)

Page 81 of 103 WhatsApp: 0954482350 / 0965038377 / 0974794056 OR EMAIL: laurentmsnd@gmail.com


LAURENT MUSONDA Call / Text: 0954482350
SUCCESS IN SCIENCE P2, 2015 – 2023 G.C.E DON’T POST / SHARE IT ON SOCIAL MEDIA

C2 (a) Calcium carbonate; and Hydrochloric acid


(b) The calcium carbonate is continuously added to dilute hydrochloric acid whilst
stirring until the carbonate is in excess. The excess carbonate is then filtered off. The
salt solution is then heated (evaporated) until a saturated solution is obtained. The
saturated solution is cooled to grow salt crystals. The crystals are then filtered,
washed and dried between two pieces of filter paper
(c) (i) Precipitation method
(ii) Pb2+ (aq) + 2 I- (aq) → PbI2 (s)

C3 (a) (i) The catalystic converter has two compartments marked A and B.
In compartment A, nitrogen monoxide is converted into nitrogen gas and
oxygen gas. In compartment B, carbon monoxide is combined with oxygen
from compartment A forming a harmless gas carbon dioxide.
(ii) 2NO (g)  N2 (g) + O2 (g) OR 2CO (g) + O2 (g)  2CO2 (g)
(b) Carbon monoxide, it reacts with the haemoglobin in blood, and prevents it from
carrying oxygen around the body – so you die from oxygen starvation.
(c) (i) By using more air (oxygen) during combustion
(ii) Design car engine which run at lower temperature.

ANSWERS FOR SCIENCE PAPER 2 – 1 AUGUST 2019 G.C.E


SECTION A [20 MARKS]
1. B 6. D 11. C 16. B
2. C 7. D 12. A 17. C
3. C 8. B 13. D 18. B
4. C 9. B 14. A 19. A
5. D 10. D 15. C 20. B

SECTION B [45 Marks]


B1 (a) (i) Helium
(ii) Carbon monoxide and nitrogen
(b) Hydrogen gas
(C) The relative molecular mass is twice larger than the vapour density.
(d) Under diffusion medium – ammonia must be in liquid state and carbon dioxide in
gaseous state
B2 (a) Simple distillation
(b) (i) A dish (bucket)
(ii) A – Round bottomed flask,
B – Liebig condenser
(c) Ethanol

Page 82 of 103 WhatsApp: 0954482350 / 0965038377 / 0974794056 OR EMAIL: laurentmsnd@gmail.com


LAURENT MUSONDA Call / Text: 0954482350
SUCCESS IN SCIENCE P2, 2015 – 2023 G.C.E DON’T POST / SHARE IT ON SOCIAL MEDIA

B3 (a) 2Al(s) + 3 CuSO4 (aq) Al2(SO4)3 (aq) + 3Cu(s)


(b) 2Al(s) + 3 CuSO4 (aq) Al2(SO4)3 (aq) + 3Cu(s)
Al(s) + 3 Cu2+ + 3SO2-4 (aq) 2Al 3+ + 3SO2-4(aq) + Cu(s)
Al(s) + 3 Cu2+(aq) 2Al3+(aq) + Cu(s)
B4 (a) (i) sodium hydroxide/calcium chloride
(ii) Calcium oxide
(iii) Aluminium hydroxide
(b) (i) Deliquescent substance are substance that absorbs enough water
vapour from the atmosphere to form a solution
(ii) Deliquescent substances are substances that absorb enough water
vapour from the atmosphere to form a solution while hygroscopic
substances are substances that absorb less water vapour from the
atmosphere and fail to form a solution.
(iii) Efflorescent substances are substances that lose water of crystallization
when they are exposed to the atmosphere.

B5 (a) Mg (s) + 2 HCl (aq)  MgCl2 (aq) + H2 (g)


(b) (i) n = V ÷ Vm = 0.24 ÷ 24 = 0.01 mol
(ii) Mass of hydrogen gas mass of magnesium used
m = n x Mr Mg  H2
= 0.01 x 2 24 = 2
m = 0.02g x = 0.02
x = (24 x 0.02) ÷ 2 = 0.24g

B6 (a) (i) Diagram B.


(ii) It is in powdered form i.e the size particles of reactants are very small.
(b) Temperature/concentration/pressure
(c) A catalyst lowers the activation energy

B7 (a) Transition metals/block D elements


(b) (i) They have high densities;

Page 83 of 103 WhatsApp: 0954482350 / 0965038377 / 0974794056 OR EMAIL: laurentmsnd@gmail.com


LAURENT MUSONDA Call / Text: 0954482350
SUCCESS IN SCIENCE P2, 2015 – 2023 G.C.E DON’T POST / SHARE IT ON SOCIAL MEDIA

(ii) They have high melting and boiling points;


They are good conductors of heat and electricity;
They are solids at room temperature and pressure except mercury which is
a liquid;
They are ductile i.e. they can be drawn into wires;
They are malleable i.e. they can be hammered into thin sheets.
(c) (i) Tungsten is used to make filament in electric bulb
(ii) Iron is used as a catalyst in the manufacturing of ammonia.

B8 (a) Corrosion is the chemical wearing of metal resulting from attack by


atmospheric oxygen in the presence of moisture.
(b) Potassium
(c) (i) Presence of moisture (water vapour); (ii) Presence of oxygen
(d) (i) Greasing and oiling; (ii) galvanizing; Painting

B9 (a) Allotropes are elements in different physical forms but in the same state.
(b) (i) Graphite (ii) Diamond
(c) Graphite is used as lubricant, this is because the layer of carbon atoms slide
over each other easily.

SECTION C [20 Marks]


C1 (a) (i) Metal X, because its metal nitrate decomposes into metal nitrites
and oxygen only.
(ii) Metal W, because its metal nitrate decomposes into nitrogen dioxide and
oxygen gas

(b) Metal X

(c) (i) Metal X, because its metal nitrate decomposes into metal nitrites

and oxygen only.

(ii) Metal W, because its metal nitrate decomposes into nitrogen dioxide and
oxygen gas.

Page 84 of 103 WhatsApp: 0954482350 / 0965038377 / 0974794056 OR EMAIL: laurentmsnd@gmail.com


LAURENT MUSONDA Call / Text: 0954482350
SUCCESS IN SCIENCE P2, 2015 – 2023 G.C.E DON’T POST / SHARE IT ON SOCIAL MEDIA

(d) Oxygen relight the growing splint.

C2 (a) (i) 2NaOH (aq) + H2SO4 (aq)  Na2SO4 (aq) + 2H2O (i)
(ii) 2NaOH (aq) + H2SO4 (aq)  Na2SO4 (aq) + 2H2O (i)
2Na+ (aq) + 2OH- (aq) + 2H+ (aq) + SO42- (aq)  2Na+ (aq) + SO42- (aq) + 2H2O (l)
2OH- (aq) + 2H+ (aq) )  2H2O (i)
(b) n = CV = 2.0 x 0.04 = 0.08 mol
(c) n = 2 x 0.08 = 0.16 mol
m = n x mr = 0.16 x 40 = 6.4g
(d) Percentage purity = (mass of pure ÷ mass of impure) x 100%
= (6.4 ÷ 8.0 ) x 100
Percentage purity = 80%

C3 (a) K – Hydration.

L – Oxidation

M – Esterification

(b) C2H4 (g) + H2O (g)  C2H5OH (g)

(c) Common name – Acetic acid.

Chemical name – Ethanoic acid

(d) CnH2n+1COOH or CnH2O2

(e) (i) Used to make perfumes.

(ii) Used in food flavouring

Page 85 of 103 WhatsApp: 0954482350 / 0965038377 / 0974794056 OR EMAIL: laurentmsnd@gmail.com


LAURENT MUSONDA Call / Text: 0954482350
SUCCESS IN SCIENCE P2, 2015 – 2023 G.C.E DON’T POST / SHARE IT ON SOCIAL MEDIA

ANSWERS FOR SCIENCE PAPER 2 – 2020 G.C.E


SECTION A [20 Marks]
1. B 6. A 11. A 16. A
2. C 7. D 12. B 17. D
3. A 8. B 13. C 18. A
4. B 9. B 14. C 19. D
5. B 10. C 15. C 20. D

SECTION B [45 Marks]


B1 (a) To condense water vapour to water liquid.
(b) Water
(c) Turn milky or form white precipitates
(d) (i) Carbon
(ii) Hydrogen
(e) Add the colourless liquid to white anhydrous copper (II) sulphate powder if it turn
blue, then the liquid is water. OR
Add the colourless liquid to white blue cobalt chloride paper if it turn pink, then the
liquid is water
B2 (a) N

Page 86 of 103 WhatsApp: 0954482350 / 0965038377 / 0974794056 OR EMAIL: laurentmsnd@gmail.com


LAURENT MUSONDA Call / Text: 0954482350
SUCCESS IN SCIENCE P2, 2015 – 2023 G.C.E DON’T POST / SHARE IT ON SOCIAL MEDIA

(c) Chromatogram is sprayed with locating agent such as silver nitrate solution which is
changed by sugar to brown spots of finely divided silver.
(d) Ink from the false bank notes and the pure ink from a genuine bank notes are run
on one chromatogram. If the distance travelled by the ink from the false bank note
is not the same as that of the genuine ink, then the notes are false.

B3 (a) V is in period 2 and group 5


(b) As the nuclear charge increases, the electrons are strongly attracted to the nucleus.
(c) R
(d) C
(e) B2E
B4 (a) (i) Concentration is the amount of solute in a known volume of a solution or
in 1dm3 of solution.
(ii) mol/dm3
(iii) Mol = concentration x volume = 0.25 x 0.05 = 0.125ml
Mass = mol x Mr = 0.125 x (39 + 16 + 1) = 7g
(b) (i) 2KOH (aq) + H2SO4 (aq) -------> K2SO4 (aq) + 2H2O (l)
(ii) 2K + (aq) + 2 OH - (aq) + 2H+ (aq) + SO42- (aq) -------> 2K + (aq) + SO42- (aq) + 2H2O (l)
2K + (aq) + 2 OH - (aq) + 2H+ (aq) + SO42- (aq) -------> 2K + (aq) + SO42- (aq) + 2H2O (l)
2 OH - (aq) + 2H+ (aq) -------> 2H2O (l)

B5 (a) Exothermic
(b) A thermometer
(c) (i) Blue copper (II) sulphate because become pale blue and finally colourless.
(ii) Brown / pink copper metal is seen
(iii) Magnesium powder reduces in amount.
(d) To minimise heat loss to atmosphere
(e) Mol = concentration x volume = 0.5 x 0.01 = 0.05mol

Page 87 of 103 WhatsApp: 0954482350 / 0965038377 / 0974794056 OR EMAIL: laurentmsnd@gmail.com


LAURENT MUSONDA Call / Text: 0954482350
SUCCESS IN SCIENCE P2, 2015 – 2023 G.C.E DON’T POST / SHARE IT ON SOCIAL MEDIA

B6 (a) (i) Carbon dioxide / methane / ozone / nitrous oxide / water vapour
(ii) Carbon dioxide – from combustion of carbon containing fuels.
Methane – from decaying organic matter or from natural gas
Ozone – from environmental pollutants reacting with oxygen.
Nitrous oxide – from agriculture activities and combustion of fossil fuel
Water vapour – from evaporation of water.
(b) Ozone – protect us from harmful radiation like ultra violet light. CFCs that leak into
atmosphere react with ozone, breaking it down to oxygen gas forming ozone holes
which allow more ultra violet rays to penetrate.
B7 (a) (i)

(ii) CH3COOH
(iii) CH3COOH  (1x12) + (3x1) + (1x12) + (1x16) + (1x16) + (1x1)
12 + 3 + 12 + 16 + 16 + 1 = 60mol/g
(b) (i) Carboxylic acid
(ii) They produce hydrogen ion when dissolved in water.
They react with alcohol in the presence of concentrated sulphuric acid
forming esters.
They react with metals, carbonate and bases forming salt.

Page 88 of 103 WhatsApp: 0954482350 / 0965038377 / 0974794056 OR EMAIL: laurentmsnd@gmail.com


LAURENT MUSONDA Call / Text: 0954482350
SUCCESS IN SCIENCE P2, 2015 – 2023 G.C.E DON’T POST / SHARE IT ON SOCIAL MEDIA

SECTION C [20 Marks]


C1 (a) - Oxygen gas
- Inert gases
(b) (i) Nitrogen dioxide – combustion of petrol or diesel in a car engine or
thermal power station furnaces that use diesel.
Sulphur dioxide – complete combustion of fuel containing sulphur
(ii) Bubble the gas through acidified dichromate and if it changes from orange
to green, the sulphur dioxide is present. OR
Bubble the gas through acidified potassium permanganate and if it
decolorised, then sulphur dioxide is present
(c) (i) IN RESPIRATION
C6H12O6 + 6 O2  6CO2 + 6H2O + Energy
IN RUSTING
4Fe (s) + 3 O2 (g) + 2 H2O (l)  2Fe2O3.2H2O
(ii) A bar of more reactive metal (magnesium / zinc) is attached to the steel oil
pipeline, a more reactive metal (magnesium /zinc) corrodes instead of the
steel oil pipeline because a more reactive metal (magnesium / zinc) is more
reactive than steel.
C2 (a) A acid is a compound which when dissolved in water produces
hydrogen ions as the only positively charged ions OR
An acid it is a substance that donates a proton willingly.
(b) (i) substance K – Sodium sulphate (Na2SO4)
Substance L – Calcium nitrate (Ca(NO3)2)
Substance M – Water (H2O)
Substance N – Calcium sulphate (CaSO4)
Substance O – Sodium nitrate (NaNO3)
(ii) Ca (OH)2 + 2HNO3  Ca(NO3)2 + 2H2O

Page 89 of 103 WhatsApp: 0954482350 / 0965038377 / 0974794056 OR EMAIL: laurentmsnd@gmail.com


LAURENT MUSONDA Call / Text: 0954482350
SUCCESS IN SCIENCE P2, 2015 – 2023 G.C.E DON’T POST / SHARE IT ON SOCIAL MEDIA

(iii) Na2SO4 (aq) + Ca(NO3)2 (aq)  CaSO4 (s) + 2NaNO3 (aq)


2Na+(aq) + SO4 2-(aq) + Ca2+aq) + 2NO3- (aq)  CaSO4 (s) + 2Na+(aq) + 2NO3- (aq)
2Na+(aq) + SO4 2-(aq) + Ca2+aq) + 2NO3- (aq)  CaSO4 (s) + 2Na+(aq) + 2NO3- (aq)
SO4 2-(aq) + Ca2+(aq)  CaSO4 (s)

C3 (a) (i) Metals : 1 - Aluminium.


2 – Tungsten.
Reason: because they conduct electricity when in solid state.
(ii) Mixture - Wax , Because it has a range of melting points.
(b) Compound - Sodium chloride – (NaCl)
(c) Movement: Particles vibrate more to a point where the forces of attraction become
weak and particles are free to move.
Arrangement: Particles changes from regular pattern to random and the spaces
widen (increased).
(d) Tungsten – because it has higher melting point meaning more energy is required to
break the bonds in between the atoms.

THIS WAS PREPARED BY LAURENT MUSONDA


CALL / TEXT 0954482350 / 0974794056 / 0965038377
WHATSAPP: 0965038377
EMAIL: laurentmsnd@gmail.com

Page 90 of 103 WhatsApp: 0954482350 / 0965038377 / 0974794056 OR EMAIL: laurentmsnd@gmail.com


LAURENT MUSONDA Call / Text: 0954482350
SUCCESS IN SCIENCE P2, 2015 – 2023 G.C.E DON’T POST / SHARE IT ON SOCIAL MEDIA

ANSWERS FOR SCIENCE PAPER 2 (CHEMISTRY) – 2021 G.C.E

SECTION A [20 MARKS]

1. A 6. D 11. A 16. C
2. D 7. C 12. A 17. A
3. C 8. C 13. A 18. B
4. B 9. X: 400tons 14. C 19. C
5. C 10. A 15. A 20. D

SECTION B [45 MARKS]


B1 (a) (i) Wash/pour/drink a lot of water.
(ii) wash the wound with spirit then tie on it with sterilized banded.
(b) To allow oxygen gas to react with methane gas so that methane gas burn
completely and produce great blue flame.
(c) (i) Carbon dioxide gas and water
(ii) pollutant: Carbon dioxide gas
Effect: Depletes the ozone layer leading to global warming.

B2 (a) Fractional distillation


(b) The glass beads provide a large surface area for condensation and allow efficient
separation of the components in the mixture.
(c) The Liebig condenser is kept in the slanting position to avoid the distillate formed by
condensation from running back into the fractionating column; and also to ensure
that cold water completely surrounds the inner glass tube where the vapour passes
in order to provide maximum cooling and avoid loss of vapour.
(d) Mixture 1: Crude oil
Mixture2: mixture of Ethanol and water

Page 91 of 103 WhatsApp: 0954482350 / 0965038377 / 0974794056 OR EMAIL: laurentmsnd@gmail.com


LAURENT MUSONDA Call / Text: 0954482350
SUCCESS IN SCIENCE P2, 2015 – 2023 G.C.E DON’T POST / SHARE IT ON SOCIAL MEDIA

B3 (a) Mg (s) + 2 HCl (aq) ---------> MgCl2 (aq) + H2 (g)


2
(b) moles of magnesium = 24 = 0.083 mols
𝐶𝑉 2 ×0.1
Moles of Hydrochloric acid = = = 0.1 mols
2 2

Therefore, the limiting reagent is magnesium.


2×2
(c) Mass of hydrogen gas = = 0.17g
24
𝑚 0.17
Moles of hydrogen gas = 𝑀𝑟 = =0.085mols
2

Volume of hydrogen = moles x molar volume


= 0.085 x 24 = 2.04dm3

52 48
B4 (a) Relative atomic mass = (100 × 107) + (100 × 109)

= 55.64 + 52.32 = 107.96 a.m.u


(b) Isotopes are atoms of the same element having the same number of protons but
different number of neutrons or mass number.

(c) (i) Carbon −14 isotope is used in radio carbon dating machine to determine
the age of fossils.
(ii) Iodine – 131 isotopes are used in medicine to diagnose and treat cancers of
the thyroid gland.
B5 (a) (i) The colour of the solution will change from reddish –
brown to pale yellow.
(ii) 2KBr (aq) + F2 (g) -------> 2KF (aq) + Br2 (g)
(b) Observation: nothing, no reaction or no colour changes.
Reason: Because bromine is more reactive than iodine hence bromine cannot be
displaced by iodine from its compound.

Page 92 of 103 WhatsApp: 0954482350 / 0965038377 / 0974794056 OR EMAIL: laurentmsnd@gmail.com


LAURENT MUSONDA Call / Text: 0954482350
SUCCESS IN SCIENCE P2, 2015 – 2023 G.C.E DON’T POST / SHARE IT ON SOCIAL MEDIA

B6 (a) The rate of reaction is the measure of how long the chemical
reaction will take place. Or it is the change in the concentration of reactants or
products in a given period of time.
(b) (i) Because an enzyme lowers the activation energy in the chemical reaction.
(ii) This is due to the fact that there are more particles in a given volume and
hence, the frequency of effective collision increases.
(iii) Smaller the size particle, increases the surface area of the reacting particles,
hence increasing the rate of chemical reaction

B7 (a) zinc blende


(b) Coke
(c) It causes breathing problem to human beings and it combine with rain water forming
acid rain.
(d) Zinc is used to galvanize iron objects zinc resists rust.
Zinc is used for making alloy called brass.
B8 (a) Poly(butene)
(b) (i) Butene
(ii)

(c) (i) Alcohol


(ii) General formula of alcohol CnH2n + 1OH

Page 93 of 103 WhatsApp: 0954482350 / 0965038377 / 0974794056 OR EMAIL: laurentmsnd@gmail.com


LAURENT MUSONDA Call / Text: 0954482350
SUCCESS IN SCIENCE P2, 2015 – 2023 G.C.E DON’T POST / SHARE IT ON SOCIAL MEDIA

SECTION C [20 MARKS]


C1 (a) (i) Calcium oxide (CaO)
(ii) Limestone is put into a lime kiln then the kiln is heated and limestone is
broken down to lime and carbon dioxide gas by thermal decomposition.
𝐻𝑒𝑎𝑡
CaCO3 (s) ⇔ CaO (s) + CO2 (g)
Thereafter, Lime (CaO) is collected.

(b) (i) Lime is used in making steel from iron.


Lime is used to neutralise acidity in soil.
Lime is used as a drying agent in industry.
(ii) More carbon dioxide gas will be produced and absorb heat in the
atmosphere, and prevent it from escaping into space causing global
warming which led to drought, melting land-ice, storms, floods, and
wildfires.
Some of the possible solutions are:
Many people are trying to cut back on using fossil fuel, for switching to clean
ways to get electricity, such as windpower and solar power, to reduce the
amount of carbon dioxide entering the atmosphere. For example, by
capturing it from power station chimneys, and burying it deep underground.

C2 (a) The leaves were pounded and chlorophyll squeezed out into a beaker. The
chlorophyll was concentrated by removing excess water. A thick concentrated spot
of chlorophyll was made on the strip of the filter paper. Acetone was put in a
beaker and the end of the strip was made to just touch the acetone with the spot of
chlorophyll just a few millimeters above the acetone. As the acetone solvent moved
up the strip, two colours namely yellow and blue were seen on the strip, yellow
colour was closer to the solvent and blue colour was above the yellow colour.
(b) (i) Sodium chloride (NaCl) because it is very soluble in water.

Page 94 of 103 WhatsApp: 0954482350 / 0965038377 / 0974794056 OR EMAIL: laurentmsnd@gmail.com


LAURENT MUSONDA Call / Text: 0954482350
SUCCESS IN SCIENCE P2, 2015 – 2023 G.C.E DON’T POST / SHARE IT ON SOCIAL MEDIA

(ii) Use acid-base titration by which strong acid reacted with strong base. The
indicator color change, pH range and the average titre values were
determined for each type of acid-base titration. These values were
comparable to those obtained from three standard indicators: methyl
orange, methyl red and phenolphthalein. Total flavonoids (TF) and
condensed tannin (CT) from the crude leaves extract were determined
which might be the major reasons for the activity of the extract as an
indicator for simple acid-base titration.

C3 (a) (i) Propene


(ii)

(iii) Alkene
(iv) CnH2n

(b) (i) Incomplete combustion will take place producing carbon monoxide which
can cause suffocation.
(ii) C3H6 + 3O2 -------> 3CO + 3H2O

(c)

Page 95 of 103 WhatsApp: 0954482350 / 0965038377 / 0974794056 OR EMAIL: laurentmsnd@gmail.com


LAURENT MUSONDA Call / Text: 0954482350
SUCCESS IN SCIENCE P2, 2015 – 2023 G.C.E DON’T POST / SHARE IT ON SOCIAL MEDIA

ANSWERS FOR SCIENCE P2 (CHEMISTRY) – 2022 G.C.E


SECTION A [20 MARKS]
1. A 6. B 11. D 16. A
2. D 7. C 12. A 17. D
3. C 8. B 13. C 18. D
4. D 9. A 14. C 19. D
5. D 10. C 15. C 20. D

SECTION B [45 MARKS]

B1 (a) Diffusion is the movement of particles from the region of higher concentration to
the region of lower concentration of the same substance down the concentration
gradient.

(b) Particles of ammonia

(c) (i) at C

(ii) Because ammonia will move twice faster than hydrochloric acid move.

(d) (i) Molecular mass of particles (size particles)

(ii) Conduct experiment in a fume hood.

B2 (a) Sugar or Limestone


(b) Brass
(c) Air
(d) Petroleum

B3 (a) (i) KJ
(ii) J2

(b) In (a)(i) KJ is ionic bonding and in (a)(ii) J2 is covalent bonding.

(c) It is high melting and boiling points


It is soluble in water
It conducts electricity in molten or aqueous state.
It is non – volatile

Page 96 of 103 WhatsApp: 0954482350 / 0965038377 / 0974794056 OR EMAIL: laurentmsnd@gmail.com


LAURENT MUSONDA Call / Text: 0954482350
SUCCESS IN SCIENCE P2, 2015 – 2023 G.C.E DON’T POST / SHARE IT ON SOCIAL MEDIA

B4 (a) It produces hydrogen ion as the only positive charged ion.

(b) (i) Weak acid is acid which partially ionise in water.


(ii) Strong acid is acid which completely ionise in water.

(c) (i) H2SO4(aq) -------------> 2H+ (aq) + SO42- (aq)


(ii) CH3COOH (aq) --------------> CH3COO-(aq) + H+(aq)

B5 (a) Zn (s) + 2HCl (aq) ---------> ZnCl2 (aq) + H2 (g)

(b) (i) moles of HCl


200
Moles of HCl = CV = 2 x 1000 = 0.4mols

Moles of Hydrogen gas produced


HCl --------> H2
2 --------> 1
0.4 --------> x

1 × 0.4
x = = 0.2mols of hydrogen gas
2

(ii) Volume = moles x molar volume = 0.2 x 24 = 4.8dm3

(c) Zn ------> 2HCl AND MASS OF HCl = n x Mr = 0.4 x 36.5 = 14.6g


65 --------> 73
X --------> 14.6
949
73x = 65 x 14.6 ------> x = 73 = 13.0g of Zinc

B6 (a) (i) W or X
(ii) V

(b) 2 protons

(c) (i) T
(ii) W

(d) UW2

B7 (a) ZnS

(b) Solid L is Zinc Oxide and gas M is Carbon monoxide gas.

(c) ZnO(s) + C (s) ---------> Zn(s) + CO(g)

Page 97 of 103 WhatsApp: 0954482350 / 0965038377 / 0974794056 OR EMAIL: laurentmsnd@gmail.com


LAURENT MUSONDA Call / Text: 0954482350
SUCCESS IN SCIENCE P2, 2015 – 2023 G.C.E DON’T POST / SHARE IT ON SOCIAL MEDIA

(d) it combines with water rain forming an acid rain which wear out the cuticle from
plant leaves causing them to wilt (dry)

B8 (a) Hydrocarbon is a binary compound of carbon and hydrogen only.

(b) (i) Alkane


(ii) Butane

(c)

(d) C10H22

SECTION C [20 MARKS]


C1 (a) (i) Sulphuric acid
(ii) Substance B is sodium hydroxide and
Substance C is magnesium sulphate

(b) (i) H2SO4(aq) + 2NaOH(aq) ---------> Na2SO4(aq) + 2H2O (l) OR


(ii) 2H+(aq) + 2OH-(aq) ---------> 2H2O (l)
(c) Neutralisation reaction

C2 (a) (i) Alkali is soluble bases that produce hydroxide ions in aqueous solution as
the only negatively charged ions.

(ii) Limestone is heated in a kiln and decomposes into calcium oxide and carbon
dioxide. Carbon dioxide is blown off, and water is added to calcium oxide of
which calcium hydroxide is formed.
(b) It is used to neutralise acidity in soil, and in lakes affected by acid rain
It is used for flue gas desulfurization.
(c) (i) CO2 (g) + Ca(OH)2 (aq) ---------> CaCO3 (s) + H2O (l)
(ii) Calcium carbonate
(iii) Calcium hydroxide (Limewater) turns cloudy or milky.

Page 98 of 103 WhatsApp: 0954482350 / 0965038377 / 0974794056 OR EMAIL: laurentmsnd@gmail.com


LAURENT MUSONDA Call / Text: 0954482350
SUCCESS IN SCIENCE P2, 2015 – 2023 G.C.E DON’T POST / SHARE IT ON SOCIAL MEDIA

C3 (a) (i) structure of propene

(ii) Propene has one double bond between carbon atoms while propane has
only single bonds between carbon atoms.

(iii) Alkene

(iv) Molecular formular of propane – C3H8


Molecular formular of propene – C3H6

(b) (i) Propene has a double bond which is rich with electrons where bromine
molecules goes and combine (reacts) with while propane has only single
bonds hence it has no where bromine molecules to combine (reacts) to.

(ii) Brown bromine will be turn to colourless.

(c) Cracking

(d) Polypropene

ANSWERS FOR SCIENCE PAPER 2 – 2023 G.C.E


SECTION A [20 MARKS]
1 2 3 4 5 6 7 8 9 10
D D C C A B C B B A

11 12 13 14 15 16 17 18 19 20
B D A D A A C A A C

SECTION B [45 MARKS]

B1 (a) (i) Matter is any substance that occupies space and it has mass.
(ii) Molecules
(b) (i) Some by-products are harmful to the environments/animals and plants
(ii) They make the environment dirty
(c) Chances of an accident occurring in the absence of a teacher are very high

Page 99 of 103 WhatsApp: 0954482350 / 0965038377 / 0974794056 OR EMAIL: laurentmsnd@gmail.com


LAURENT MUSONDA Call / Text: 0954482350
SUCCESS IN SCIENCE P2, 2015 – 2023 G.C.E DON’T POST / SHARE IT ON SOCIAL MEDIA

B2 (a) Low temperature makes the particles in a gas enclosed in a balloon lose kinetic
energy and they start moving slowly and close to each other hence the decrease in
volume.
(b) An exothermic process
(c) On a warmer day temperature is higher and this increases the Kinetic energy of the
particles in a liquid on the cloth hence this increases the rate of evaporation and in
turn clothes dry faster than on a cold day-where temperature is low and the rate of
evaporation is slow.

B3 (a) A mixture of sand and iodine crystals can be appropriately separated by heating the
mixture and the iodine crystals will sublime and the sand will be left behind on an
evaporation dish.
(b) Bottled mineral water is not pure water because it has mineral salts and air that are
dissolved in it making it to be a mixture not a pure substance.
(c) - Boiling point - Melting point -Freezing point -Density

B4 (a) (i) Q - Chlorine


(ii) Period 3, because it has 3 shells
(iii) It gains one electron / Shares one electron
(b) Nucleon number is the sum of proton and neutrons in the nucleus of on atom.

B5 (a) An acid produces hydrogen ions as the only positively charged ions when dissolved
in water while a base produces hydroxide ions as the only negatively charge ions
when dissolved in water.
(b) A salt is a compound which is formed when the ionisable or replaceable hydrogen
ion of an acids completely or partially replaced by a metal or Ammonium ion.

B6 (a) A mole is an amount of substance which contains as many elementary particles as


the carbon atoms in 12g of carbon 12 isotope.
250
(b) 𝑛 = 𝐶𝑉 = 0.25 × 1000 = 0.0625mols

(c) mass of Fe2O3 : mass of CO


160 : 84
3 200 : 𝑥
3 200 × 84
160 𝑥 = 3 200 × 84 → 𝑥= = 1 680kg mass of carbon monoxide
160

Page 100 of 103 WhatsApp: 0954482350 / 0965038377 / 0974794056 OR EMAIL: laurentmsnd@gmail.com


LAURENT MUSONDA Call / Text: 0954482350
SUCCESS IN SCIENCE P2, 2015 – 2023 G.C.E DON’T POST / SHARE IT ON SOCIAL MEDIA

B7 (a) Metal occupy a high position in the reactivity series form very stable carbonates as
a result some of them like Na2CO3 do not decompose while some like MgCO3 don't
easily decompose while metals that occupy a low position in the reactivity series
form unstable carbonates like CuCO3 hence they easily decompose on heating.

(b) (i) A catalyst is substance that alters or changes the rate of a chemical reaction
without itself chemically changing.
(ii) Metals are good conductors of heat and electricity because they have
delocalised electrons which are free and mobile and can carry heat and
electrical energy.

B8 (a) Carbon trichlorofluoride (CCl3F) Reason: It has the largest greenhouse factor, i.e.
21 000
(b) This is because methane has a much higher greenhouse factor which is 30, while
carbon dioxide has a greenhouse factor of 1 which is less.
NB Candidate must quote figure from the table.
(c) The depletion of the Ozone layer which act as a protective layer against harmful
rays that can cause
(i) Skin cancer or (ii) Cataract of the eye

B9 (a) (i) 𝐶𝑛 𝐻2𝑛 + 2

(b) It produces small useful hydrocarbons


It prolongs the life-span of natural crude oil reserves
Because they are chemically unreactive

C1 (a) (i) Bubbles of colorless gas seen in the test tube / effervescence
Plunger is pushed outwards
(ii) Carbon dioxide gas
(iii) Test: Bubble the gas in limewater in a test tube.
Result: limewater turns milk (white precipitate formed).

Page 101 of 103 WhatsApp: 0954482350 / 0965038377 / 0974794056 OR EMAIL: laurentmsnd@gmail.com


LAURENT MUSONDA Call / Text: 0954482350
SUCCESS IN SCIENCE P2, 2015 – 2023 G.C.E DON’T POST / SHARE IT ON SOCIAL MEDIA

(b) (i)

Volume against Time


70

60

50
Volume (cm3)

40

30

20

10

0
0 20 40 60 80 100 120 140
Time /s

(ii) (70, 75)

𝑉𝑜𝑙𝑢𝑚𝑒 65
(iii) Average rate of reaction = = 90 = 0.72cm3/s
𝑇𝑖𝑚𝑒

C2 (a) Zinc sulphide

(b) Roasting stage: The zinc ore is roasted in air to produce zinc oxide
2 ZnS(s) + 3 O2 (g) → 2 ZnO(s) + 2 SO2 (g)
Reduction of Zinc oxide: Zinc oxide is reduced by coke and zinc metal and carbon
monoxide are produced
ZnO(s) + 𝐶 (s) → Zn(s) + CO (g)

Page 102 of 103 WhatsApp: 0954482350 / 0965038377 / 0974794056 OR EMAIL: laurentmsnd@gmail.com


LAURENT MUSONDA Call / Text: 0954482350
SUCCESS IN SCIENCE P2, 2015 – 2023 G.C.E DON’T POST / SHARE IT ON SOCIAL MEDIA

(c) - Galvanising iron -To make alloys e.g. brass


- To make dry cells -To make medicine

(d) - Land degradation - Deprives organism of habitat


-Soil erosion - Land slides - Pollution

C3 (a) (i) Both burn readily in oxygen to give carbon dioxide and water/stem

(ii) Ethane does not react with hydrogen but ethene undergoes hydrogenation
in the presence of nickel or platinum catalyst.

(iii) Equations in (a) (i) Ethane: 2C2H6 (g) + 7O2 (g) --------> 4CO2 (g) + 6H2O(g)
Ethene: C2H4 (g) + 3O2 (g) --------> 2CO2 (g) + 2H2O(g)

𝑁𝑖 𝑜𝑟 𝑃𝑡
Equations in (a) (ii) Ethene: C2H4 (g) + H2 (g) → C2H6(g)

(b) Ethane Ethene

(c) Test: Put equal volumes of aqueous bromine in two separate test tube and bubble
each of the alkanes into the bromine solution.
Result: Ethene decolourises bromine readily while ethane does not.

THIS WAS PREPARED BY LAURENT MUSONDA


CALL / TEXT 0954482350 / 0974794056 / 0965038377
WHATSAPP: 0965038377
EMAIL: laurentmsnd@gmail.com

THE END

Page 103 of 103 WhatsApp: 0954482350 / 0965038377 / 0974794056 OR EMAIL: laurentmsnd@gmail.com


LAURENT MUSONDA Call / Text: 0954482350

You might also like